{ "cells": [ { "cell_type": "markdown", "id": "3960a901", "metadata": {}, "source": [ "# Centre Universitaire de Mila" ] }, { "cell_type": "markdown", "id": "c4568d4a", "metadata": {}, "source": [ "## Master 1 (STIC & I2A), Matière: Apprentissage Automatique" ] }, { "cell_type": "markdown", "id": "a8bedc4a", "metadata": {}, "source": [ "## Travaux pratiques N°6 : Machine à vecteurs de supports" ] }, { "cell_type": "markdown", "id": "f57e8c63", "metadata": {}, "source": [ "## Enoncé" ] }, { "cell_type": "markdown", "id": "af6bff48", "metadata": {}, "source": [ "# Setup" ] }, { "cell_type": "code", "execution_count": 41, "id": "a0b23fb3", "metadata": {}, "outputs": [], "source": [ "import sys\n", "import sklearn\n", "import numpy as np\n", "import os\n", "\n", "# to make this notebook's output stable across runs\n", "np.random.seed(42)\n", "\n", "# To plot pretty figures\n", "%matplotlib inline\n", "import matplotlib as mpl\n", "import matplotlib.pyplot as plt\n", "mpl.rc('axes', labelsize=14)\n", "mpl.rc('xtick', labelsize=12)\n", "mpl.rc('ytick', labelsize=12)" ] }, { "cell_type": "markdown", "id": "88db096e", "metadata": {}, "source": [ "# 1. Classification SVM linéaire\n", "Vous pouvez considérer qu'un classificateur SVM correspond à la rue la plus large possible (représentée par les lignes pointillées parallèles) entre les classes. C'est ce qu'on appelle la classification à grande marge.\n", "\n", "### Question:\n", "Appliquer le classificateur SVM linéaire sur le dataset Iris." ] }, { "cell_type": "code", "execution_count": 42, "id": "dead834d", "metadata": {}, "outputs": [ { "data": { "text/plain": [ "SVC(C=inf, kernel='linear')" ] }, "execution_count": 42, "metadata": {}, "output_type": "execute_result" } ], "source": [ "from sklearn.svm import SVC\n", "from sklearn import datasets\n", "\n", "iris = datasets.load_iris()\n", "X = iris[\"data\"][:, (2, 3)] # petal length, petal width\n", "y = iris[\"target\"]\n", "\n", "setosa_or_versicolor = (y == 0) | (y == 1)\n", "X = X[setosa_or_versicolor]\n", "y = y[setosa_or_versicolor]\n", "\n", "# SVM Classifier model\n", "svm_clf = ...(kernel=\"linear\", C=float(\"inf\"))\n", "svm_clf.fit(..., ...)" ] }, { "cell_type": "code", "execution_count": 43, "id": "9a7b3b5d", "metadata": {}, "outputs": [ { "data": { "image/png": "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\n", "text/plain": [ "
" ] }, "metadata": {}, "output_type": "display_data" } ], "source": [ "# Bad models\n", "x0 = np.linspace(0, 5.5, 200)\n", "pred_1 = 5*x0 - 20\n", "pred_2 = x0 - 1.8\n", "pred_3 = 0.1 * x0 + 0.5\n", "\n", "def plot_svc_decision_boundary(svm_clf, xmin, xmax):\n", " w = svm_clf.coef_[0]\n", " b = svm_clf.intercept_[0]\n", "\n", " # At the decision boundary, w0*x0 + w1*x1 + b = 0\n", " # => x1 = -w0/w1 * x0 - b/w1\n", " x0 = np.linspace(xmin, xmax, 200)\n", " decision_boundary = -w[0]/w[1] * x0 - b/w[1]\n", "\n", " margin = 1/w[1]\n", " gutter_up = decision_boundary + margin\n", " gutter_down = decision_boundary - margin\n", "\n", " svs = svm_clf.support_vectors_\n", " plt.scatter(svs[:, 0], svs[:, 1], s=180, facecolors='#FFAAAA')\n", " plt.plot(x0, decision_boundary, \"k-\", linewidth=2)\n", " plt.plot(x0, gutter_up, \"k--\", linewidth=2)\n", " plt.plot(x0, gutter_down, \"k--\", linewidth=2)\n", "\n", "fig, axes = plt.subplots(ncols=2, figsize=(10,2.7), sharey=True)\n", "\n", "plt.sca(axes[0])\n", "plt.plot(x0, pred_1, \"g--\", linewidth=2)\n", "plt.plot(x0, pred_2, \"m-\", linewidth=2)\n", "plt.plot(x0, pred_3, \"r-\", linewidth=2)\n", "plt.plot(X[:, 0][y==1], X[:, 1][y==1], \"bs\", label=\"Iris versicolor\")\n", "plt.plot(X[:, 0][y==0], X[:, 1][y==0], \"yo\", label=\"Iris setosa\")\n", "plt.xlabel(\"Petal length\", fontsize=14)\n", "plt.ylabel(\"Petal width\", fontsize=14)\n", "plt.legend(loc=\"upper left\", fontsize=14)\n", "plt.axis([0, 5.5, 0, 2])\n", "\n", "plt.sca(axes[1])\n", "plot_svc_decision_boundary(svm_clf, 0, 5.5)\n", "plt.plot(X[:, 0][y==1], X[:, 1][y==1], \"bs\")\n", "plt.plot(X[:, 0][y==0], X[:, 1][y==0], \"yo\")\n", "plt.xlabel(\"Petal length\", fontsize=14)\n", "plt.axis([0, 5.5, 0, 2])\n", "\n", "plt.show()" ] }, { "cell_type": "markdown", "id": "3bcbc9b0", "metadata": {}, "source": [ "# 2. Cas non séparable de SVM\n", "Pour éviter les problèmes des cas non séparables de SVM, utilisez un modèle plus flexible. L'objectif est de trouver un bon équilibre entre garder la rue aussi large que possible et limiter les violations de marge (c'est-à-dire les instances qui se retrouvent au milieu de la rue ou même du mauvais côté). C'est ce qu'on appelle la classification des marges souples.\n", "\n", "Lors de la création d'un modèle SVM à l'aide de Scikit-Learn, nous pouvons spécifier le nombre d'hyperparamètres. C est l'un de ces hyperparamètres.\n", "\n", "### Question:\n", "Appliquer le classficateur SVM linéaire pour le cas non séparable sur le dataset Iris en choisions les valeurs de C à 1 et 100." ] }, { "cell_type": "code", "execution_count": 47, "id": "47f7c7bc", "metadata": {}, "outputs": [ { "data": { "text/plain": [ "Pipeline(steps=[('scaler', StandardScaler()),\n", " ('linear_svc', LinearSVC(C=1, loss='hinge', random_state=42))])" ] }, "execution_count": 47, "metadata": {}, "output_type": "execute_result" } ], "source": [ "import numpy as np\n", "from sklearn import datasets\n", "from sklearn.pipeline import Pipeline\n", "from sklearn.preprocessing import StandardScaler\n", "from sklearn.svm import LinearSVC\n", "\n", "iris = datasets.load_iris()\n", "X = iris[\"data\"][:, (2, 3)] # petal length, petal width\n", "y = (iris[\"target\"] == 2).astype(np.float64) # Iris virginica\n", "\n", "svm_clf = Pipeline([\n", " (..., StandardScaler()),\n", " (..., LinearSVC(C=1, loss=\"hinge\", random_state=42)),\n", " ])\n", "\n", "svm_clf.fit(X, y)" ] }, { "cell_type": "code", "execution_count": 48, "id": "d90a3922", "metadata": {}, "outputs": [ { "name": "stderr", "output_type": "stream", "text": [ "C:\\ProgramData\\Anaconda3\\envs\\tf-gpu\\lib\\site-packages\\sklearn\\svm\\_base.py:1208: ConvergenceWarning: Liblinear failed to converge, increase the number of iterations.\n", " ConvergenceWarning,\n" ] }, { "data": { "text/plain": [ "Pipeline(steps=[('scaler', StandardScaler()),\n", " ('linear_svc',\n", " LinearSVC(C=100, loss='hinge', random_state=42))])" ] }, "execution_count": 48, "metadata": {}, "output_type": "execute_result" } ], "source": [ "scaler = StandardScaler()\n", "svm_clf1 = LinearSVC(C=..., loss=\"hinge\", random_state=42)\n", "svm_clf2 = LinearSVC(C=..., loss=\"hinge\", random_state=42)\n", "\n", "scaled_svm_clf1 = Pipeline([\n", " (\"scaler\", scaler),\n", " (\"linear_svc\", svm_clf1),\n", " ])\n", "scaled_svm_clf2 = Pipeline([\n", " (\"scaler\", scaler),\n", " (\"linear_svc\", svm_clf2),\n", " ])\n", "\n", "scaled_svm_clf1.fit(..., ...)\n", "scaled_svm_clf2.fit(..., ...)" ] }, { "cell_type": "code", "execution_count": 49, "id": "0846169c", "metadata": {}, "outputs": [], "source": [ "# Convert to unscaled parameters\n", "b1 = svm_clf1.decision_function([-scaler.mean_ / scaler.scale_])\n", "b2 = svm_clf2.decision_function([-scaler.mean_ / scaler.scale_])\n", "w1 = svm_clf1.coef_[0] / scaler.scale_\n", "w2 = svm_clf2.coef_[0] / scaler.scale_\n", "svm_clf1.intercept_ = np.array([b1])\n", "svm_clf2.intercept_ = np.array([b2])\n", "svm_clf1.coef_ = np.array([w1])\n", "svm_clf2.coef_ = np.array([w2])\n", "\n", "# Find support vectors (LinearSVC does not do this automatically)\n", "t = y * 2 - 1\n", "support_vectors_idx1 = (t * (X.dot(w1) + b1) < 1).ravel()\n", "support_vectors_idx2 = (t * (X.dot(w2) + b2) < 1).ravel()\n", "svm_clf1.support_vectors_ = X[support_vectors_idx1]\n", "svm_clf2.support_vectors_ = X[support_vectors_idx2]" ] }, { "cell_type": "code", "execution_count": 50, "id": "619929c7", "metadata": {}, "outputs": [ { "data": { "text/plain": [ "(4.0, 5.9, 0.8, 2.8)" ] }, "execution_count": 50, "metadata": {}, "output_type": "execute_result" }, { "data": { "image/png": "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\n", "text/plain": [ "
" ] }, "metadata": {}, "output_type": "display_data" } ], "source": [ "fig, axes = plt.subplots(ncols=2, figsize=(10,2.7), sharey=True)\n", "\n", "plt.sca(axes[0])\n", "plt.plot(X[:, 0][y==1], X[:, 1][y==1], \"g^\", label=\"Iris virginica\")\n", "plt.plot(X[:, 0][y==0], X[:, 1][y==0], \"bs\", label=\"Iris versicolor\")\n", "plot_svc_decision_boundary(svm_clf1, 4, 5.9)\n", "plt.xlabel(\"Petal length\", fontsize=14)\n", "plt.ylabel(\"Petal width\", fontsize=14)\n", "plt.legend(loc=\"upper left\", fontsize=14)\n", "plt.title(\"$C = {}$\".format(svm_clf1.C), fontsize=16)\n", "plt.axis([4, 5.9, 0.8, 2.8])\n", "\n", "plt.sca(axes[1])\n", "plt.plot(X[:, 0][y==1], X[:, 1][y==1], \"g^\")\n", "plt.plot(X[:, 0][y==0], X[:, 1][y==0], \"bs\")\n", "plot_svc_decision_boundary(svm_clf2, 4, 5.99)\n", "plt.xlabel(\"Petal length\", fontsize=14)\n", "plt.title(\"$C = {}$\".format(svm_clf2.C), fontsize=16)\n", "plt.axis([4, 5.9, 0.8, 2.8])\n" ] }, { "cell_type": "markdown", "id": "751221fd", "metadata": {}, "source": [ "# 3. Classification SVM non linéaire\n", "Malgré que les classificateurs SVM linéaires soient efficaces et fonctionnent très bien dans de nombreux cas, de nombreux datasets ne sont pas linéairement séparables. Une approche pour le traitement des datasets non linéaires consiste à générer des attributs, telles que des attributs polynomiales; dans certains cas, cela peut aboutir à un dataset linéairement séparables.\n", "\n", "### Question:\n", "Générer l'ensemble de données (dataset) moons. Afficher ces données à l'aide d'un graphe 2D." ] }, { "cell_type": "code", "execution_count": 51, "id": "d901fd77", "metadata": {}, "outputs": [ { "data": { "image/png": "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\n", "text/plain": [ "
" ] }, "metadata": {}, "output_type": "display_data" } ], "source": [ "from sklearn.datasets import make_moons\n", "..., ... = ...(n_samples=100, noise=0.15, random_state=42)\n", "\n", "def plot_dataset(X, y, axes):\n", " plt.plot(X[:, 0][y==0], X[:, 1][y==0], \"bs\")\n", " plt.plot(X[:, 0][y==1], X[:, 1][y==1], \"g^\")\n", " plt.axis(axes)\n", " plt.grid(True, which='both')\n", " plt.xlabel(r\"$x_1$\", fontsize=20)\n", " plt.ylabel(r\"$x_2$\", fontsize=20, rotation=0)\n", "\n", "plot_dataset(X, y, [-1.5, 2.5, -1, 1.5])\n", "plt.show()" ] }, { "cell_type": "markdown", "id": "e69cb11f", "metadata": {}, "source": [ "Pour implémenter cette idée à l'aide de Scikit-Learn, créez un pipeline contenant un transformateur PolynomialFeatures, suivi d'un StandardScaler et d'un LinearSVC. Testons cela sur le dataset qui s’appelle \"moons\": il s'agit d'un ensemble de données fictives proposé pour la classification binaire dans lequel les points de données ont la forme de deux demi-cercles entrelacés. Vous pouvez générer cet ensemble de données à l'aide de la fonction make_moons().\n", "\n", "### Question:\n", "Appliquer la classification SVM non linéaire sur le dataset \"moons\"." ] }, { "cell_type": "code", "execution_count": 52, "id": "c93abf9c", "metadata": {}, "outputs": [ { "name": "stderr", "output_type": "stream", "text": [ "C:\\ProgramData\\Anaconda3\\envs\\tf-gpu\\lib\\site-packages\\sklearn\\svm\\_base.py:1208: ConvergenceWarning: Liblinear failed to converge, increase the number of iterations.\n", " ConvergenceWarning,\n" ] }, { "data": { "text/plain": [ "Pipeline(steps=[('poly_features', PolynomialFeatures(degree=3)),\n", " ('scaler', StandardScaler()),\n", " ('svm_clf', LinearSVC(C=10, loss='hinge'))])" ] }, "execution_count": 52, "metadata": {}, "output_type": "execute_result" } ], "source": [ "import numpy as np\n", "from sklearn import datasets\n", "from sklearn.preprocessing import StandardScaler\n", "from sklearn.svm import LinearSVC\n", "from sklearn.datasets import make_moons\n", "from sklearn.pipeline import Pipeline\n", "from sklearn.preprocessing import PolynomialFeatures\n", "X, y = make_moons(n_samples=100, noise=0.15)\n", "polynomial_svm_clf = Pipeline([\n", "(\"poly_features\", PolynomialFeatures(degree=3)),\n", "(\"scaler\", StandardScaler()),\n", "(\"svm_clf\", LinearSVC(C=10, loss=\"hinge\"))\n", "])\n", "\n", "....................." ] }, { "cell_type": "code", "execution_count": 53, "id": "07f4af09", "metadata": {}, "outputs": [ { "data": { "image/png": "iVBORw0KGgoAAAANSUhEUgAAAlcAAAHFCAYAAADffdxRAAAABHNCSVQICAgIfAhkiAAAAAlwSFlzAAAPYQAAD2EBqD+naQAAADh0RVh0U29mdHdhcmUAbWF0cGxvdGxpYiB2ZXJzaW9uMy4yLjIsIGh0dHA6Ly9tYXRwbG90bGliLm9yZy+WH4yJAAAgAElEQVR4nOzdeXSb5Z03/O8t2XJsy7ZsRbJlxwtJFBKwE0hJk1DWJu3DOtMyJ2U57dsFmFOY0/ftwNtMGYa9LTlMOyVMKTMFJk5LgRYKk2cgdKbkeSkQQpqAA85CsJ14ia1YiixZ8ipZ9/3+YeR4lWTrXqXv5xwOiazlumNb/vp3XdfvEiRJkkBEREREsjBpPQAiIiKiTMJwRURERCQjhisiIiIiGTFcEREREcmI4YqIiIhIRgxXRERERDJiuCIiIiKSEcMVERERkYwYroiIiIhkZNhwFQ6HsXXrVnz5y1+Gw+GAIAh48MEHU3psY2MjBEGY9b/Tp08rO3AiIiLKaDlaD2Ch/H4/fvWrX2HNmjX4yle+gmeeeWbez7Fjxw6sXLlyym12u12uIRIREVEWMmy4qq2tRSAQgCAIOHPmzILCVX19PS666CIFRkdERETZyrDhShAErYdARERENINhw5UcrrvuOvh8PpSUlOCKK67Aww8/jPr6+oSPGR0dxejo6MTfRVFEX18f7HY7Ax8REZFBSJKEcDiMyspKmEzyLkHPynBVUVGBe++9Fxs2bEBxcTGam5uxbds2bNiwAXv37sWaNWvmfOyjjz6Khx56SMXREhERkVK6urqwZMkSWZ9TkCRJkvUZNXDmzBk4HA488MADKe8YnK69vR0NDQ344he/iF27ds15v+mVq/7+ftTU1OB3Oz9GYaFtyn2H+0YAACVl5gWNSc9EaQx90Q9Qlvs5mISFZfSxvtDEn/PL8uQamqJEKYbeaDPKcxtgEjLv8zoXXre+rlvo65txm6XMKtvzi5KIzugnqMldCZNg2E3l88brnv26/e9+gvrKPkSlXhw6tx9LzrlEg1HKL9w/iP912TcRDAZRUlIi63NnZeVqNnV1dbjkkkvw/vvvJ7xfXl4e8vJmBoHCQhushWUAgCH/MABgSbV8b3Z6I0pjGIkUoMhSOu9wNebvH/9DgQ0F9kUKjE45ojSGcKQAxRbbgkOlEfG6tbtuwe+bcZulWt7fsqcTpRgKIgUotpToKlQqjdc9+3WP5hSiZKwHsUILCq05KLYVajBK5SixpCd73iVTIElSWvOu8VAFAKX27PnGTMVEoAIMF6iI1DBbiAIAi71Y5ZEQzc5UWgQgoPUwDIHh6jMnT57E3r17sXnz5gU9frhvBIUF+QxV0zBUEc3EIEVG4G8JQOw8hQr/brSXRtBjjqDAsU7rYRmCocPVG2+8gcHBQYTDYQDA0aNH8fLLLwMArrnmGhQUFODWW2/Fzp070dbWhtraWgDA5s2bcdlll2H16tUTC9ofe+wxCIKARx55ZMHjYbA6i6GKiCGKjMvfEkDBgbdgzfkUI+UheNa7UOCogdPm1npohmDocHXHHXego6Nj4u8vvfQSXnrpJQDjlai6ujrEYjHEYjFMXrff0NCA3/3ud/jpT3+K4eFhOJ1OfPGLX8R9992HFStWLGgsmbhofb4YqChbzRWiAAYpMh7fnmZYe1tR6Q6iudSP2Koa1Lo3aT0sQzF0uGpvb096n8bGRjQ2Nk657ec//7kyA8pSDFWULViJokzn29OMOpyExR1Ec+kx5K9aA7t7rdbDMhxDhyvS3lhfCCaYGaooY0wOUAJEwAoIfX4IMDFEUVYorSlCzGqGUOhjsFoghiuatzF/P0TEACtQUJaXVVvzKTMkmsYDzlaiRCkGRABLWVFWbc0nEsNBILM6LqiKPxUpZfHpvwL7IojSGAIRjQdElATXQhEt3Ji9QOshGBbDFSU1OVQR6Q0DFJG8xIEATEvY0yodDFc0Ky5SJz1JdRqPiEgPGK5oCoYq0hKrUETaMg/7IZQUQ8KQ1kMxNIYrAsBQRephgCLSqX6v1iPIGAxXxDVVpAiGKCJjCew7joLBMxgKBtFk64CAcq2HZFgMV1mMoYrkwBBFZGx9bUGgGrAPH0TEPYymuggK6tfxqJs0MFxlIYYqWqh4M83pGKKIjMnfEkB+0zsIVtdB3FAIT+0iniEoA4arLMJ1VZSq6dWoeKdyNtMkyhzxMwSdNT3woA6dNUAdzxCUBcNVFmCookRSOS8v3qmciDKLe50NwxgBANQsu1zj0WQOhqsMxlBF080WpDilR5S9xIEAYNV6FJmH4SpDcV0VMUgRUSpMNqYruTFcZRiGquzEIEVE8+FvCcA87MdQ6BT6orkAqrUeUkZhuMoQnALMLgxTRLRQ/pYACg68BWvOp+i/sBCdNQAGtR5VZmG4ygCsVmU2BikikoO/JQCx8xSsva3IqTqIjqooYnWFqFl6OU58zB0rcmK4MjCGqszEMEVEcosHq4rBdzDsDsJ7bgksK1fCaXNDjElaDy/jMFwZEKcAMwvDFBEpTew8hTqcRPG6WhwsHMAiexXsbBSqGIYrg2G1yvgYpohIC6U1RYhhCABgd6/VeDSZjeHKIBiqjG16oGKYIiItiOEgUKj1KDIfw5UBMFgZj5phyuM1IxCa+0ia0uIYXM6YYq9PRMYyZi/QeggZj+FKxxiqjEWL6pTHa8a1t1UiEp15mPLEOHJFvP5MDwMWUZYTBwIwLSkCENB6KBmP4UqnGKz0Tw9TfYGQOWGwAoBI1IRAyMxwRUSkEoYrnWGo0jehzw9YP/s/TFw7RUSGYB72az2ErMJwpSMMVvo0uUIV7wZjKSuCSZh7nRMRkV74WwIQAAglxfBEPtF6OFmB4UoHGKr0Z64pP1GKAWxkTEQG4W8JQDj+ERxD+3EyGIFnfSnyHTVaDyvjMVxpjMFKP/SwhoqISC6+Pc2w9rbCmvMp2jYPQCgvR76jBk42D1Ucw5VG2GVdHxioiCjTxI+6qcNJBD87Q7Cg/hKGKhUxXGmA1SrtTQ5VDFRElEniwcpS7J9yhiCph+FKZQxW2snEQFVaHIMlV0za56q0mG0YiLJJaU0RolIYABisNMBwpRJOA2oj06f9XM4YXn+mhx3aiWhW7MauDYYrFbBapb5MrFLNxeVkeCKiqcSBAM8Q1BDDlcIYrNSTTYGKiCgZUymPutEKw5WCGKzUwVBFRHQWu7Frj+FKAQxV6mCoIiKanVBSDAlDWg8jazFcyYzBSlkMVEREpHcMVzJisFIOQxURUWLxo26WFAVwMngYPbYIChzrtB5WVmK4kslYXwgosDFYyYyhiogoOX9LAAUH3oI151MEK8fPECzgUTeaYbiSEYOVfBiqiIhSM/kMwY61fsRW1fAMQY0xXMkkvyxP6yFkhHioYqAiIkrOt6d5/KgbdxDNpX5YLuMZgnrAcEW6wFBFRLQw8aNuBOdiBiudYLgiTTFUZTaP18yjeYhUwqNu9IPhijTBUJX5PF4zrr2tMumh0q8/08OARUQZZe53PSIFCH4fg1WWCITMCYMVAESipoSVLSJKThwIfHbUDekFK1ekCu7+IyKibMFwRYpiqCIiUg7PEdQnhitSBEMVEZE6eI6g/jBckey4poqISHm+Pc0o8rVjKDiGJlsHBJRrPST6DMMVyYbVKiIi5flbAhA7T6HCvxvDayJoqougoH4de1zpCMMVpY2hiohIHRNnCFo9GCkPwbPexTMEdYjhitIi9PkBmBiqaIbS4hgsuWLSPlelxexxRZSK+BmCle4gmkvbITgXo9a9Seth0SwYrmhBhD4/YB3/M4MVzcbljOH1Z3rYoZ1IRu51NsSsFgiFPlRtuF7r4dAcGK5o3gS/D9Jnf7aUsXEdzc3lZHgiouzDcEUpm7wLUJRiQETjARERZSExHMRYDc8R1DMef0MpYXsFIiKi1LByRQkxVJFeebxmrueirCMOBIBCrUdByTBc0ZwYrEivPF4zrr2tMulOxNef6WHAoowzfkhzQOthUAIMVzQD+1bpEys1ZwVC5oTBCgAiURMCIXPW/JsQkX4wXNEUrFbpEys1RMRDmo2DC9ppAoOVfs2nUkNEmUso4fuzERg2XIXDYWzduhVf/vKX4XA4IAgCHnzwwZQf7/V68a1vfQuLFy9GQUEBNm7ciD179ig3YB0T/D4Ifh8s9mIGKyIiojQZNlz5/X786le/wujoKL7yla/M67Gjo6PYtGkT9uzZg+3bt2PXrl0oLy/HVVddhT//+c8KjVifWK0iItI/Idir9RBoHgy75qq2thaBQACCIODMmTN45plnUn7ss88+i8OHD+O9997Dxo0bAQBXXnkl1qxZg61bt2L//v1KDVtXGKyIiPTP3xKAcPwY8gfP4GTwGHpsEQgo13pYlIBhK1eCIEAQhAU99tVXX8W55547EawAICcnB1//+tfxl7/8Bd3d3XINU7cYrIiI9M/fEoDYeQqOof0wu4+hpy6Cgvp1PLBZ5wxbuUrH4cOHcemll864ffXq1QCAI0eOoKqqatbHjo6OYnR0dOLvoVAIACBKMYjSmAKjlZfQN77bJPezMwFFaWE7y+KPW+jjjUqr60719ca/DuUfm94+3yVFEVhyxaS7J0uKImmNWW/XrRZetz6uu68tiPymd1BY2ItofQ56Vhcjf/ESLC5aBjEmJX+CFMWfS87nNAIlrzcrw5Xf70dZWdmM2+O3+f1zb3d99NFH8dBDD824vTfajHDEAGc9WT/7v0znArZHj8rzRAaj9nV3R0sALEnhfq1YFOlXbBy6+XzbgCd/eRyhkGXOuxQXRzBsG8YJGb7WdXPdKuN1a6waCFbXAagb//sgEB4Ewgod7Np+JKrI8+rV0JBy15uV4QpAwinFRB+75557cNddd038PRQKobq6GuW5DSi22GQdo5ziFSvLZxWrdIlSDO3Ro6jLPQ8mIXu2/2t13fl2c0qVmnr7OXBZlKlc6e3zvbQKwOwF5inSab6qx+tWA69b++s+8+cjWL3Ej1jhMJoKj6Fy3dWKvZYYk9B+JIq683NhMi9suY0RhYIMV7Ky2+2zVqf6+voAYNaqVlxeXh7y8vJm3G4SzDAJ+vznHF9fZVJkfdX4dWfPm2+c2tddVQ68/kxPCiEBAJQbl9E+3x6vGdffnn7zVaNdt1x43doRYgJyJECABAExVUKPySxkVbhS8lr1mQYU1tDQgObm5hm3x2+rr69Xe0iK4cJ14+ExN/LhMTlkdGI4iLEaAyw5oSmyMlx99atfxZ133on9+/dj/fr1AICxsTE899xzWL9+PSorKzUeoTzijUHJOHjMjbEwCBPRbAwdrt544w0MDg4iHA4DAI4ePYqXX34ZAHDNNdegoKAAt956K3bu3Im2tjbU1tYCAL7zne/gySefxJYtW7Bt2zY4nU788pe/xPHjx/Hmm29qdj1yYbXKuFhpMQ4GYSKai6HD1R133IGOjo6Jv7/00kt46aWXAAAnT55EXV0dYrEYYrEYJOnslsu8vDzs2bMHW7duxfe+9z0MDQ3hggsuwBtvvIHLL79c9euQE4MVzVcq1ZdyB8PBdAzCpDRxIAAUaj0KWghDh6v29vak92lsbERjY+OM28vLy7Fz5075B6UhBquZ9DxtM9vYTnSp+y2ZavXlv57uAvS7GZYoY5lKiwAEtB4GzZOhwxWdxWA1k56nbVIZmxrmU31ZpHC40nMQJiKaD4arDMBgNTs9T9ukMrZsoucgTEQ0X3x3NzgGK8oE8wnC81FaHIMlV0x4H0uuiNJiBjbSF/Pw3CeFkP6xcmVgDFZEibmcsRSbrzJckf4IJcWQMKT1MGgBGK4MisFKWx6vGf3huc+1U/IHdm6OhGh2HQGWFpeT4YmI1MVwZUAMVtry+fLxd3dWa7Y+KDom4Nv/UM71RxqLTzkm+zrglCNR9mG4MhgGK+2FQhbNF8qzf5L2OOVIShGCvVoPgdLEcGVADFYkl/lUX4ZVHJdRcMqRlNB3MohYkQ3AGZyOdGo9HFoAhisD4VmBqYn3SzrTZxpfnzQ298nnWk3bpBJq1JBq9aXcEcOJiIoDI8pS/pYAhG4Pyk0f42SwH571pch31Gg9LJonhiuDiE8HUmKp9EvKyRHxxH0+LC4TNZu2SRRqTnTl4IePOVQdS7J/A1FK+OG06XH9ksdrhr/fjO5oCUZyLTAJUz9XnPIjufn2NMPa2wprzqdo2zwAobwc+Y4aOG1urYdG88RwZQBcZ5W6VPoljY2ZsLhMxHnLtS3FcErpLL2tX5oa0pfMep94SHfXRfl5pLT4WwIQO0+hDicRcB9DR2kQBfWXMFQZGMOVzjFYZYfJR7+ofb6gXugpbKYa0u98oJyd4ylt8WBlKfbDd24O8u1rYGewMrTsfBc3CAar7KCXcwZpYbhzk+RQWlOEmNUMwAe7e63Ww6E08d1cpxis9Ku4OCLrkSoLOWeQ/ZOIiPSLlSsdY7BSTnzqbSHreByOYfzX012qd2jfttWHpdVjij0/ERHJg+FKh7gzUHnx3XgLXS/jcsZQVa7ugvil1WOaL8InIqLkOC2oM5wOVFd8vQwREZFcWLnSEQar9OmlOWem8XjNaPPP3u8J4DSlnk3eiTobfu70QwwHMVZToPUwSAYMVzrDYJWeyf2S1G7Gmak8XjOuv70SkWjtnPfJtHYEmRLSU9mJmmmfOyI9MPY7RwbhOiv5uJwxnLc8MrH4m9KTym7GTJtejYf0XzzoQU5O4tCh552b2fi5I9IDVq50gNOB2U2PR7/QeMAqd0Tw1FN7UDR83qzToQCn1YhoJoYrnWCwyl56O/olE8i5zsjhGMZSS2TOcEVENB3DlcYEv4/BinR19IvRcZ0REWmNa640xHVWyopPtyXC6bbM09Key3VGZDjiQEDrIZCMWLnSGKtWyuF0W/bxeM34fx5xaj0MogUxlRYBYMjKBAxXGuF0oDoyebqN/YtmCoTMiI4JWg+DiLIcw5UGOB2ovEwPHmquK1JrN2Omf860wJ2oRNpguNIIq1bKyYYFzfPpX5TuNbqcMfzX01047D+JqtzlinRoz4bPmRY4NU6kDYYrlbFqpTw1g0e2cDljGLb1K9aSgJ8z5WTy1DiRXnG3oIrYLFRfzvTxy5+IiOTHypXKGKz04/9+xIE3nuU0UzbKycm8dUZcs0akHwxXKuF0oP6MjXGaKVs9cZ8voz7vXLNGpC8MVypi1Yqy0b7+d/Fo+4O4p+5BbCy5RNHXSnV3nLsumvB54lUgUYqhO1qCkVzLlLVmeqsCcc0akb4wXKmAVSvKVpIk4fGux3BipBWPdz2GDcVfgCAo14dKjt1xM6tAS2bch1UgkpN52K/1EEhmDFcqYdWK5GSU/kXv9b+NI4MfAwCODH6M9/rfxhdslyv6munujmMViLQglBRDwpDWwyCZMFwpjFUr9ZUWx5CbI2V0p24j9C+SJAlPnPoZTDBBhAgTTHji1M9wccllM6pXRgmLRESpYLhSAatW6nI5Y9h+nxd3PlCu9VAUpff+RZOrVgAgQpyzemWEsEhElCqGKwWxaqWdxWWi1kPIatOrVnGJqld6CIserxknuvi2SMawr6cJj77/FO7ZcAc2Vl6o9XBoEnZRVBirVtqITzMlwmkm5cSrVpODFTC1eqU38YXsP3zMofVQiJKSJAmPH9yBE/1dePzgDkiSpPWQaBL+iqYQVq20xWkm7cSrVgIESJj5hi9AmLN6paVUFrLrwWzNQs/0mZCTI2JsjGvWssV7PR/iiL8FAHDE34L3ej7EF6o+p/GoKI7hSkGsWmlLD9NM2SgqRXB6tGfWYAUAEiT0RjyIShFYhDyVR2dsqTQLzc2RsP0+74ypcf4ykTkkScITH+yESTBBlESYBBOe+GAnLq5cq6tfWLIZw5UCWLUivfG3BNJ6fOny1H9RsJjy8GL9/0ZgrG/O+5Tl2GExGTdYaVUFSqW6Fh0TsLhMxHnLIyqNitQ2uWoFAKIksnqlMwxXCmHVivTC3xKA2HkKrpqFf7v3/LkL2Jj6/V15lXDlVS749fRs21YfPnf+KKtApInpVas4Vq/0heFKZqxakZ749jTD2tsKq9WDomD+gp/HORhBCOvR1xbE4uV2GUdoPEurxxisSDPTq1ZxrF7pC8OVAli1Iq3Fq1V1OIlg1UF0VEUhOBcv+PnEsiCA9SjY9zp8HRfCVLMEdnepfAMmoqTiVauEm0VYvdIFhisig0i2bso04J34c+GhfSgs7EWwPATvhSWwrFwJp8294Nfu7WvFaAdgPqcNFT0e+IbXIzDgmnE/0epM+TkZzojmJypGcXrQl3izyNAZRMUoLGaLyqOjyRiuZCT4faxakSLi03uLy8e34EvD/TPvVGSd+ONIeQhtGwYglLuQ76hJK1gBgKNkGcKIwHNFBc7piGHxRx9jUWha2AsPzHickF8y6/Od6Y3Bjyt0FbB4BA/pncVswYvXP4HAyCzf/58pyy9hsNIBhisiHYtP79UOH0J/1ScYWvxZWLEVzHLvs2+4J/N9KKhfl3aomq5m2eVoF/4PpHwfzhlOoSdU0DP170VFAABzcQAFBwBf53LdTDHO1htNlGLojraiKnc5TIKZ7QxIcy6rAy4rG93qHcOVTIS+PqCAVSuST6Dp+MT0Xn95CD2bx6tQqShA+tWqudS6N8HraEHvPB9n9oUn/jxiC8OBY6jweHC681L4oY9pwum90UQphkWRfiy1RGAS5m5IqxZW14iMgeFKRpwSpIXw7Wme+LN52D/x5/zBM3CtGUNT3fj0Xq17kxbDm9WCgpvt7B+9jiL48AmWFAyg4shuDPSu0N00oR7x5AEiY2C4ItLI5Ck/W1XhxO1CSTGGgu3ohQdNdRFFpve05rS5gQ1utNv3wJzXifLugyh8tx99x+sgnbuGISsBnjxApH8MV0Qa8LcEUPjurokpv2Hn5J13PUA54KkrRYEMi9H1LD7F2OPrhGt/FxytvfAdBwIDLpReeK7WwyMiWhCGK5lYyqzJ70QEoO+1t1Doa0fF0hAONZzd0TddbQaHqsmcNje8ADzrAamuF0vf/xhDh9rR1+2B/ZJVkGzlWg+RiGheGK6IFBCf8pssvp7KMbQfw2siOJShU34L4bS5AZsbHdiDExt6Udnej/wWK/zvHgNwDAAQyz/bGV4vOwyJiGbDcEUkM39LAAUH3oLV6kGxa/zIGWkoDBQBQlEBguWF8NQVZvyU30LEpwk95Z1w4RiWYXy6VAoPTdwn3J+jqx2GRETTMVwRySRerbL2tqLSHURTrRe5i8b7UY3Z432pRgBAVzv/9GbyNKHns38vYLz1QI5/CJL3DGo/DGOgdwV8ncvh2NSg2ViJiGbDcEUkg3iwqhh8B8PuIJpLgyiqWoPFRUshVdiSPwFNEZ8mnMENeIMt6MC7KO8+iLoxM9r3cJqQiPSF4YooTZOnAcULBPhsOci3r4HdvXaOE8AoHU6bG97LAO8nnwBN+1HRewyncY2hpwk9XjN7VxFlEIYrogWaPA1odh9DR2kQgm0xltRdymqVwpw2N7wrgR57J8zHOlH74W7DThN6vGZce1tl0q7rrz/Tw4BFZBAMV0QL0NcWhNTZMzEN6C0NIn8Vq1Vqik8deh3GniYMhMwJgxUARKImBEJmhisig2C4IlogV00OioL5kJaXIN9ZC7t7rdZDykrxacIeXyfw5vg0IY/TISItpXCsvT4NDAzg+9//PiorK7Fo0SJccMEFePHFF5M+rrGxEYIgzPrf6dOnVRg5ZRJpaPww4pijSOORqG9fTxP+6pW/xb6eppRuV5LT5ka+owY9m13oWOuHNedTFBx4a8q5jUREajFs5eqGG27AgQMHsG3bNqxYsQLPP/88br75ZoiiiFtuuSXp43fs2IGVK1dOuc1ut89xb6JZDIw3BZXs2ResJEnC4wd34ER/Fx4/uAMbrr8AgiDMebsaMmWakIiMz5Dhavfu3fjTn/40EagA4Morr0RHRwd+8IMf4MYbb4TZPPfOGwCor6/HRRddpMZwdYM7kuQjDPpgCfkgFBUkv3MGeq/nQxzxtwAAjvhb8F7Ph/hC1efmvF1NM3YT9p/CwAEXpwlJl4RgL2JFNgBntB4KyciQ4erVV1+F1WrFli1bptz+7W9/G7fccgv279+Piy++WKPR6RN3JMkj8HELsBJY1HoEwXOOoqcuAsE8glpb9jQFlSQJT3ywEybBBFESYRJMeOKDndjounDW2y+uXKta9SouvpvQi08gedtRexTAgbcMuZuQiIzHkOHq8OHDWLVqFXJypg5/9erVEx9PFq6uu+46+Hw+lJSU4IorrsDDDz+M+vr6pK89OjqK0dHRib+HQiEAgCiJECX9hhJ/f2o7kvz9QLkj+XXEr1XP1yy3vrYg0HsaWGmD4D6BnrWLkb94CRwlyyDGMnuPYPz6xJiEfd1nq1PA+Nf+EX8Lnv7od7Pe/m7XB6pXrwBgcdFyYN1y9LV9hHahGY7gAKrDOeh8S4JpSRXKliVvl6HG13mqzy1KMdW+37Lx+xvQ6rpFSGZgTBAgQoAkmVV/P5n8/Z1NlLxeQ4Yrv9+PpUuXzri9rKxs4uNzqaiowL333osNGzaguLgYzc3N2LZtGzZs2IC9e/dizZo1CV/70UcfxUMPPTTj9s7oJyiI6HeKqDtaAmBJCvdrxaJIf8rP2x49CgDw+fIRClnmvF9xcQQOx3DKz6tL1QCqx38g95z7JWAQCA8CYUS0HZeKTh6O4Gef7oQJJogQJ243wYRfffQ7CBAgTWpGYYIJP3tvJypW1KtevTprFbBkFU4tAcaP0hYBdCEY6Ur5GeJf50oI5+cjN9eFaHTuKfvc3BjC+UdxIqLu95CS161nql53PoB1wF7YAdiBwWqc+Fib95T2I1FNXlcrQ0PKXa8hwxWAhG/UiT521VVX4aqrrpr4+2WXXYZrr70WDQ0NuP/++7Fr166Er3vPPffgrrvumvh7KBRCdXU1anJXothSMo8rUNdI7tzBZ7Kq3OVYakn+jS1KMbRHj6Iu9zz0+iz4uzurk045/tfTXYabcuxrC0I81Q1r1yGYGrrRUyNCLLwWdefnwmTWKiyoT4xJaD8ShaesGdNvCYsAACAASURBVK3DrTM/DhGj0uist7cOt+L04sOaVK+m6O1HT+d7qOwvgs2bh9bwhUkrWJO/zk1C4nWcC7W0Cnjt6VMprIdcrsjrz0aN69YjTa6734szh3vRUOWHaB3CAdcp1Cy7XJ3X/kz8+zvb3tdCQYarKex2+6zVqb6+PgBnK1ipqqurwyWXXIL3338/6X3z8vKQl5c343aTYNL1m1CqYzMJ5nldh0kwoz9sSWnKsT9sQVW5cao8/pYApM4euPxvYKQmhJ51LhTaaxHuAExmIePehPb1NOHR95/CPRvuwMbKC2d8XJIkPPnxb2ZUp5IRIOAXh36NS6o/p2H1CkClDfmDFciJAebTEVRW5aJXSu37dr7fF/NVVQ5UlSf7xUP99xelr1uv1LxuASYIMSBHkiBCgiDENHtvycT3tUSUvFZD9rlqaGjAsWPHMDY2NuX25ubxnjaprJ2aTpIkmEyG/OcgBfj2NKPgwFuo8O9Gx1o/eja7kO+ogaNkmdZDU8T0FgqSNDM8jUljOD3om1ewAgAJEnqHziAqaj/lEO9HFu9PRkSkBENWrr761a/i6aefxh/+8AfceOONE7fv3LkTlZWVWL9+/bye7+TJk9i7dy82b94s91ApBbO1iDjTZ0JocDzsFheKWFwmTvm4Um0jJp8XmFN1EB1VUcRW1aDWPb4bMFMXfKbSQiHXlIsXrt2OYCQ04/FnhgMIjQ6gJM8Ke/7Mdgdl+SWwmFObmlbaeF+yuddlEhGly5Dh6uqrr8aXvvQl3HHHHQiFQli+fDleeOEF/PGPf8Rzzz030ePq1ltvxc6dO9HW1oba2loAwObNm3HZZZdh9erVEwvaH3vsMQiCgEceeUTLy8pKqbSImI2SbSNcNTkoKg6if/l5sJxfNN6cMoPN1VphthYKFYUOVBY7NRqpPNrNnXAV9SO/6X9DtF8DP8D+V0QkK0OGKwB45ZVXcO+99+L+++9HX18fVq5ciRdeeAE33XTTxH1isRhisdiUKY6Ghgb87ne/w09/+lMMDw/D6XTii1/8Iu677z6sWLFCi0tRRWlxDJZcMemi89JidRecp3Jo7WyUPsh2fNooOzr2T65aAWdbKGjRAFRpTpsbXgCe9UBl2IOK3t08h5CIZGfYcGW1WrF9+3Zs3759zvs0NjaisbFxym0///nPFR6ZPrmcMbz+TA87tKcii461mV61itOyAajS4sfkdGAPzMc6Ud/dBQ8bjBKRjAwbrmj+XE6Gp1Rly7E206tWcZlcvYqrdW+C19GCw2+/C2fBAOpCPIeQiOTB7XGUtviUYyJaTDnOl29PMwrf3QWpZS+a6jrQbu7UekiKiletBMxemRIg4IkPds66czBTOG1u5K9ag0XLa2Ep9sNVw983iSh9fCehtBl9yjG+Q7AOJxFc2oK2BhMK6tdl/EL2qBhN2FphcguFHOSqPDr1xBxFgJetGYhIPgxXJAujTjn6WwIoOPAWrFYPgiXd6NnsQoGjJuODFQBYzBa8eP0TCIzMfdxRvIVCpragiJPsRZA+bgeSHzdIRJQUwxVlrYmeVlYPxGUD8FaXID9LglWcy+qAy+rQehj6MeAHrBVaj4KySN/JIABgKNiOYN6IxqMhuTBckaZSaRExG7nWcLlqclAUzEd/ZQlyV2Z+TyuaW7ZsYtDabE2DJ9PzEgK5+VsCKDy0DwXlIfQW9cNTV4p8R43WwyIZMFyRpuZar6VVh3Yi04AXAHcLKiGVpsFKNgjWE9+eZlh7W1GxNIRDDT4I5eUTJ0GQ8S0oXA0ODmLp0qXwer0455xzcPz4ceTmzlzwOjIygs2bN2Pv3r2wWCz47//+b1xxxRXpjpkyjNbrtaShMCR7nWavTzpSYgPmXoJGaUqlabDSDYK1Fl+OUDH4DobdQRwuDaKg/hJWzTPMgloxFBYW4h//8R8BjJ/LN71RJzC+zfsb3/gG9u7dC0EQsHPnTgYrg/F4zTjaapnzP58vX+shEski3nbD0tUCqduDQNNxjUdEmSq+M7nYlQ/P+lLkr1qjarDa19OEv3rlb7Gvp0m111SLnq5twdOC3/3ud/Ev//Iv6OzsxI9//GN885vfhMVy9mDWu+++Gy+//DIA4J//+Z+nHEtD+pdK+T4314XXnj6FqnIVBya3AR7gm+3iR+I01R1AZbsHiwdLcKYb8FudKF1erPXwVBVfDyVKMXRHSzCSa4FJODtlz+l4eZTWFCFmNQPoxOKipXM0Q5GfJEl4/OAOnOjvwuMHd2DD9RdkzAkMeru2BYervLw83H///bjtttvQ0dGB//iP/8B3v/tdAMD27dsnjpn5/ve/j7vvvlue0ZJqUinfR6Pjb8RV5cZ+s+VCZnLa3PDWA57yTtS1A469+zH4bjv6hOuBaq1Hp46Zv1AtmXGfbFkPpZacYBjSSvX6f0w+kSHTTmDQ27Wl1aH9W9/61sRhxz/5yU8QiUTwyiuv4K677gIAbNmyBT/72c/SHyURkcKcNjdq3ZvQXge0bR6AeWkL8j98BwDQ1xbUdnDTJJuy93jn3o03l/mshyLjmXyOKHD2/NBMOIFBj9eW1m5Bs9mMhx9+GDfddBO6urpw55134re//S1EUcRll12G3/zmNzCZeMIO6dP4rjCiqeJnDlo8n6DM3A0P6hR/zfm0J+COO1qI6eeITj4/dGPF2nk9176eJjz6/lO4Z8Md2Fh5odxDnbdE16ZV9SrtVgxf+9rXsG3bNhw6dAjPPvssAOD888/Hrl27kJeXN+tjuru78dJLL2H37t345JNPcPr0aZSVleELX/gCtm7divXr16c7LKLUlNgAfRUlSC8aVgJvdyn+MvMNS9xxR/M1ubIjSmfb2sQrPBuuST0g6W1tU7Jru7hyrSbjS7usJAgCbr/99om/O51OvPHGG7DZ5p5H/td//Vf8/d//PU6cOIEvfelLuPvuu3HJJZdg165duPjii/H73/8+3WERERlCNk3HZcoh70YTr+xMDh/A1ArPfJ8LwLwfqwQ5r01OaVeuWlpa8MADD0z8fXBwcM6KVdznP/95vP3227j00kun3P7OO+9g06ZNuOOOO/DXf/3XSZ+HaKGEYC+A8a33KNJ4MKRbgpXtRuRk9EPejShe2REgzHpIuwABvzj0a/xoyWMpP1e8SqR1dSiVa9NqfGlVrrxeL6666iqcOXMGdrsdwHi4+vGPf5zwcTfccMOMYAUAl156Ka688kr09fWhubk5naERJeR/9xgWfdqMvCI/muo60G7uZBM/IhW4nDGctzwy538MVvKKilGcHvTNGj4AQIKE04NnMCaNJX2u6VUiratDqVxb79AZRMWoyiNLo3I1ODiIa6+9FidOnIDVasX//M//4JFHHsF//ud/4t///d9x9913o6Zm/mckxTu95+TwZB4tpXLmX25uzHDl+4nuyEP7MeyOoKkugoL6dQxWlJAw6ANg13oYlEHEcBAoVP51LGYLXrz+CQRG5j56wGYpwVDbzFNWJtPj2qZUrq0svwQWs2XOjytlQQlmbGwMW7ZswcGDB5GTk4Pf//73WLt2LR566CHs2rULo6OjeOihhyYWuKeqs7MTb775JioqKtDQ0LCQoZFMkpXvRSmGcP5RuJzLARhjLYi/JYCCA2/BavVgpDwEz3oXChw1DFZESO0XKq6HkteYXZ0eey6rAy6rY+Lv03f7iTEJJxBJ+BzTd+TFab0zb/q16cWCwtV3v/tdvPHGGwCAp556CldffTUAYPXq1fibv/kbvPzyy9i5cyf+4R/+YaIPVjLRaBTf+MY3MDo6isceewxmszF+YGeyRGf+iVIMJyLDKo9o4eIVq0p3EMFyAV5bCQ9JJZpk8i9U4x3aW1GVu5wd2jPMbLv9UnmMXtc26dW811w9+OCDExWp++67D7fddtuMj5tMJsRiMdx3330pPacoivjOd76Dt99+G7fffju+8Y1vzHdYREm5asZ/lxCKipG7cqXGoyE9c9rcaDd34qOa8VYM+R//BX2vvQV/S0DjkSm7427yeqhly/q5HioDLWS3n57XNunVvCpXzz77LB566CEAwDe/+U08/PDDM+5z/vnn42tf+xpefPFFvPTSS7jnnntwwQVzJ2NJknD77bfjueeew9e//nX827/92zwvgYhIfrXuTei1tyLcAZgauuH41IPTnXb4AdjdpbK9znyn47jjjhZqrt1+yfpc6Xltk16lHK527949cXbg5s2b8fTTT8953wceeAAvvfQSYrEY7r33Xrz++uuz3k8URdx2223YsWMHbr75ZjQ2NrKjO5FOTF6Xsb48+dRBJnKULEMYEZy+yIYc60nUHn0HAwdc8HUuh2OTPOtCFxKWEk3ZE80lUSdzFxJ/Pet1bZNepRyurrnmGkSjqZX8Vq5cibGxxNs6JwerG2+8Eb/5zW+4zopIJ6avy3j+mse1HpKmapZdDk9fGOLoACqHgvg0NL6OT64KFsNSdhEHAqrsFJws0W6/VPtcUeo0KROJoohbb70VO3bswJYtW/Dcc88xWBHpiN66MOtB7sqVECqrAJxdv0e0UKZSdbsXJ+tkfih8SNXxZDpN3iEefvhhNDY2wmq1YsWKFfjRj3404z5f+cpXEq7VIiJlzLYug7/ZEhlXKrv9fuv5LW6QPg+Au/3koEm4am9vBwAMDAzM2c29rq6O4UpBHq+ZC2JpVnOtyzhUcgjLwEPVpaEwMPfRqUS6k8puvzPRM4iKYzCDi9LloEm4amxsRGNjoxYvTRgPVtfeVpl0d9Lrz/QwYGWZROsy+JstINmLAPi1HgYZmHlY/a+fZLv9RFHCwMkCWMyJu7RT6rhwIAsFQuaEwQoAIlETAiEzw1WWSdSFuXW4Fe/1fIhLay7SYGREmUMoKYaEIVVfM9FuPzEm4UR34g7tND8MV0QEILV1Gb849GtcUv05dmEe8APWCq1HoStcakB0FsMVZRVpKAzJXqf1MHQplXUZpwfHuzBnc7NAoUid8+CMhEsNUuNvCUxMqp+OdGo6FlIWwxURAUi8LkMUJXS3RNHQ4MjqYEWz41KD5PwtAQjHP4JjaD9OBiPwrC9FvqNG62GRQhiuiGjCXOsyxJiERd0RVBQyWMWZBrwA5DsGhzKXb08zrL2tsOZ8irbNAxDKy5HvqIHT5tZ6aKQQhisiovkqsQFzH7NGBGC8WiV2noK1txVm9zF0lAYhlNeg1r1J66GRwniQHxFRitrN4+tkLF0tkLo9Go+G9E7sPIU6nESlOwjfuTnIX7WGwSpLMFxlodLiGCy5YsL7WHJFlBZn3toIoaAIgj+s9TDIgJw2N/IdNWiq60Cw6DCWoAt9r70Ff0tA66GRjpXWFGHREicAwO5eq/FoSC2cFsxCLmcMrz/Tw23TRPPktLnhrQc85Z3A/nY4WnswcCAAP66Q7RBnIjI+hqss5XIyPBEthNPmhheAZz1gLu5E7VEPcOAt+DqXw7GpQevhkQ6J4SDGatjCI5swXBERzZPT5gZsbvjxIcTRblQOBRG25aC3JZCVFaz4UoNkfa4ycakB0WwYrshQ2AWa9CTmKIJQWQW0ZvfRIVxqMDdxIAAUaj0KUhvDFRkGu0BTttPzLxdcajA3U2kRAG58yCYMV2QY7AJNeiUNhQGbsq/BXy6Mxzzs13oIpBGGKyKiNEj2IgDK/xDlLxfGEW8eWosutEvt6DFHUOBYp/WwSEXsc0WU5fb1NOGvXvlb7Otp0nooxjbAKgWNB6uCA2+hwr8b/UWfwLO+FAX163jUTZZh5Yom6Hk9BylDkiQ8fnAHTvR34fGDO7Dh+gsgCILWwzIcoYjb7GnSGYJWDzrO8yO2qoZnCGYphisCwPUc2eq9ng9xxN8CADjib8F7PR/iC1Wf03hUxmTpPgmce77WwyCN+PY0ow4nYXEH0VzajvxVa9iRPYtxWpAAzG89RyYw+3gEjiRJeOKDnTAJ4593k2DCEx/shCRJGo/MOJw2N9rNnWiq64ClKIDCd3fBt6dZ62GRRuJH3QjOxQxWWY7hirKPTeFtXQYRr1qJ0vg5k6IkTlSvKHW17k0oqF+HQw0+mJe2oGLwHfj2NPPMwSwlhoNaD4F0gOGKDCObD5yW2/SqVRyrVwvjtLlRUL8O3gtLMFLSjYrBd1BwgIc6Z6sxO9fgZTuuuSLDSLcLtKdzDEXFgNTViRFbGMjisv3ktVaTTa5ece3V/DhtbmCDG934LywJCqjsDeLTzlPwA7IcicMjZvTL3xKAtbcVAzld6F9u7JrFvp4mPPr+U7hnwx3YWHmh1sMxLIYrSpuauwwX2gXa7i6FH0DPgVZYPR4sWQa02/dk5U6eeNVKgAAJMytUAgQ88cFOXFy5ljsHF2CRvQpCDEBvBK6aHPTK9Lw8Ykaf4jsEXVVdOFx1CrG6GtS6N2k9rAXh7mH5MFwZgJ5bJHi8Zlx/uzF2GY4HrCsw0HkKFUd2wzUaQE9dL7z1yKqAFRWjOD3omzVYAYAECb1DZxAVo7CYLSqPzvhijiLAq8yGCR4xo719/e/i0fYH8Xe5d2OtZwnqcBIB9zEcLg3Cctklhn4v4e5h+TBc6ZzeWyQYrWu03V0KuEvhe82Pgo/acSHG0IQD6CjvNOxvm/NlMVvw4vVPIDDSP+d9yvJLGKzSpMaROKQuSZLweNdjODHSimeDP8HVgzch4OqH79wc5NvXwG7gYCVJEn5x6NcwCSaIkjix/pIV7IVhuNI5tcJLtq3nKLvuCgSajmM0dBQXtgPBxU70BlsM/VvnfLisDrisDq2HkbHUOhKH1PVe/9s4MvgxAODooi4cqR5C8eIcLKm7FFKFsZP0ofChKeswuf4yPQxXBCB713NEqt3IC/KHIClkwA9YK7QeBclAkiQ8cepnMMEEESLMEPDjvv/ENvtfGT5YSZKE33p+O1G1imP1auEYrmgC13MQyYdH4mSWyVUrAIhBwqFYN/5i9mKJhuOSw3s9H6J1uHXG7axeLZyx94wSEemcacCr9RAoTZOrVpOZIeA3He8aui9cfK2VgNkrU/Hdw0a+Ri2wckXZrZ/dlEl+7eZO1JUDwkcfIr+3GAEApReeq/WwDEGPu6OnV63iYpDQMtBr6MoOdw8rg+GKspZodQIDPq2HQRnGaXMDNjfasQdSvg+V7f3IPwT4+iIw1SyRpaFoptLj7uh41SpT+8JZzBa8cO12NDefQZU7FybTzGvg7uH547QgpYVH0hDNLn7mYE9dZPzMQf9uiJ2neCROAno8QD4qRdAT7kypsmNUFYUOLCtYhvPsy3He4vH/+iNh/PDtx9AfCaOikDuL54uVK53Te4uEbN1lSNow2tEcTpsb3nqgp7wTdYeGUHFkN07jGtmOxCFl+VsCEDtP4Q/DN+F0cQf8zjHkrr0AZdaaKffLtMoOO7Wnz7DhamBgAP/0T/+E3//+9+jr68PKlSvxwx/+EDfddFPSx3q9XmzduhWvvfYahoaGsGbNGvzoRz/Cpk36ayJphPBi9F2GUnhI6yFQCpR8w1cytDltbngBoAEoOtMOoVK+I3FIOf6WAAoOvAWr1QNxmQnR6nNQsXJlVvTCY6f29Bk2XN1www04cOAAtm3bhhUrVuD555/HzTffDFEUccstt8z5uNHRUWzatAnBYBDbt2+H0+nEk08+iauuugpvvvkmLr/8chWvIjVGDy+6ZrUDIa0HQalQ6g1fzd/ShaIC9r4ygDN/PoKinjZUuoNoLm2H4FyMqg3Xaz0sVcTPHmWn9vQYMlzt3r0bf/rTnyYCFQBceeWV6OjowA9+8APceOONMJtnr/Q8++yzOHz4MN577z1s3Lhx4rFr1qzB1q1bsX//ftWug4hSo+Qbvha/pY+3Z+C0oN70tQWBasA5uA8Rtx/NpUHkr1oDu3ut1kNTzeTvB4C9rhbKkAvaX331VVitVmzZsmXK7d/+9rfR09OTMCC9+uqrOPfccyeCFQDk5OTg61//Ov7yl7+gu7tbsXET0cLE3/Dj3aMnv+GnY3JoA852pFaip0+7uRO98MAS8kHq9iDQdFz216D0iKfG3/8da6vHzwvMsmA1/fshTsnvi0xlyMrV4cOHsWrVKuTkTB3+6tWrJz5+8cUXz/nYSy+9dMbt8cceOXIEVVVVc7726OgoRkdHJ/4eCo3PKYmSCFHKnqm7+LUa+ZolQUQMEmJmM2KSAEkExFjiN4/4x5PdL9Noed3Tq1Zx8Tf8DeUXLrh6tbf7g1l/S3+36wN8oepzsl334qLl8IkCej5vhnjwE5SOFKPvNOD/SELpav2t4dHq+zvV1xOlmCJjk8zjn+fIYBhSoRmlSy/Miu/1+DXuPTX1+2Hi49O+LzKFkp9bQ4Yrv9+PpUuXzri9rKxs4uOJHhu/33wfCwCPPvooHnrooRm3d0Y/QUEk+467aI8e1XoIC1cNBAGg+gvACIAOIIxISg9tP2Lcbdfp0OK6m0JNCd/wX9n7F1xYPP9F6JIk4Wef7pw4Ky7OBBN+9t5OVKyonwht8lx3NYBqnFoFYFX8thEEIs0yPLcy1P7+DufnIzfXhWh07g08ubkxhPOP4kRkWP4BfH78fx9UnwvgXJz4OLX3g0wgSRL+5f1fJ+znNf37wuiGhpR7PzNkuAKQ8JOb7BOfzmPvuece3HXXXRN/D4VCqK6uRk3uShRbShI+NpOIUgzt0aOoyz0PJkG9njNy6msLwoFeFPUfQP9SC7yrrHCULEv4GDEmof1IFHXn58Jkzow3mFRodd2SJOHe3c8nfMN/uf953PCFz8/7DX9v9wdo/WiW89QgonW4FacXH8bGirWKXHdn258heb1wdVogNldheHENJHcDypbp4wBgrb6/l1YBrz19KoXd0cvlf/F+L/r+chzBy21YFzqFJlcXapbpb4OTEsSYhJbmIfRJZxL28wpIflTXm2Ax56o8QmWEggxXU9jt9lkrTH19fQAwa2VKjscCQF5eHvLy8mbcbhJMhg0Z6TAJZsNetyCZYIYAsyjCDBGCSUr5B6jJLGRVuIpT+7ojsdSO5ogJY/PqMzT5PLW5QtsvDv0aF18zvt5G7uuuW3EFvM4W9JZ3wiV2wNnqge8TICCt0VX/Ky2+v6vKgaryZFN+Cx/TbE1cTQNeSN0e5I/6EYQNH5o/gEmwZ9X3eK4pFy9eux3ByNzbp8vyS7DIkjn9vJT8/BoyXDU0NOCFF17A2NjYlHVXzc3j5fX6+vqEj43fb7JUHktE6rKYLXjx+icQGOmf8z4LaeCY+nlqY/N63vmI97/yrAekul4se3M/Bt9th6/zc3BsalDsdbNZvCloHU4CAKThs19XI6YAhpYNAliGgvPWorxMgeqYzlUUOlBZ7NR6GBnBkOHqq1/9Kp5++mn84Q9/wI033jhx+86dO1FZWYn169cnfOydd96J/fv3T9xvbGwMzz33HNavX4/KykrFx086ZLPB7AsC+piV0Z33e5rw8LGncP/iO3Bxtbq7p1xWB1xWeY/fSD205QIprsNbiPg5hB3Yg7bNvahsj6DoI6DvNT+kc/VVxTK6yU1Bhy4QgHB4/AO2+FpZE07X2oBBJF0ekIhWpwgY7fSCTGfIcHX11VfjS1/6Eu644w6EQiEsX74cL7zwAv74xz/iueeem+hxdeutt2Lnzp1oa2tDbW0tAOA73/kOnnzySWzZsgXbtm2D0+nEL3/5Sxw/fhxvvvmmlpdFpEuSJGF7UyNOjZ7C9qZGbFyy8N15epJKaFNrp1itexO8jhZ4yjvhQhccrb3wHQf8YMBKV7xaZe1thdl9DB2lQQi2xVi0rAoxR9GU+9YULU9rEbtWx8bwuBr9MWS4AoBXXnkF9957L+6///6J429eeOGFKcffxGIxxGKxKb058vLysGfPHmzduhXf+973MDQ0hAsuuABvvPGGLruzk7I8nWPIgx0jBz/CcGkQXkdRVhxvMR88CkMdnCaUXzxYVQy+g5GqbnjPLUG+fe7eVemGaa2+V/g9qj+GDVdWqxXbt2/H9u3b57xPY2MjGhsbZ9xeXl6OnTt3Kjg6MgK7uxRwl6J9D2BtiaE251O0lR1AR3kn8h01DFlQ/igMTmVMxWlC+fj2NMPa2wqr1YOO88aPsMlduRJ2hb6vtTo2hsfV6JMhO7QTycmxqQFD667AwNgKLHvTCtf+AIZ9nfAGZ/ZWyjZKdUYHZk5lsPvzWbXuTSioXwfP+lKY1nTBMbQfwvGP2NU9Bf6WwESwyqk6iI7z2hFbVYOqDdcr+guTkt8renxdSozhigjjVayhdVfAV7AetnA9AxaUPwpjtqkMOstpcyPfUQPP+lK0bR7AYtPHyD+0D32vvQUh2Kv18HRpYhrQvxs5VQfRs9kFy2WXoNa9SdHX1erYGB5Xo1+GnRYkkpvdXYrAgAuRksUoD/qRF3Mim3+ETT/ANU6Og1w5lZGa+DSh19GCEziAyvZ+LAmX4NS7xwAcAwDE8u0T9zfVLMmKqUPfnpntdMzDfhT62lFY2Iu2zQMQyl2qTe8r+b2ix9el5BiuiKbrD2o9As3Fw0+iJpvphKHpPxT4wyAxp80Nbz3gKe8E9h/GsiIXAEAKD03cJ9yfg4EDLvhxRcYGrMk7/yrd49+n0tBnLRWKAG9lP9rqIhDKyxWvVgHjawZ/su+XkCAp9r0yF6W/Ryk9DFdEk4hWJzDg03oYmku9yWZ03g08kx3ErOQPAyMvoJ+8m9CDkc9uHZ8OyvEPQfKeQe2HYeAA4OtcnnE7DKfv/Dt87vhxY2P2yWe6lqJApWpVfM3gydApmAWzIt8riSj5PUrpY7giohmmN9kURQndLVFUuXNhMo0Hn4V0Rge0m8rIhF5A8WnCGdyAN9iCtrIDqGo+iNrwAE59tsMwVVpUu2Y7imY2YucpFHV+gMLCXnSs9U/s/NNyR+/kr+OYFMO9G+7EGsf4idySJOEf3/kp2vo7saykBv/25UdkDzhKnV5A8mC4OO21vwAAIABJREFUIqJZTW6yKcYkLOqOYKndktZ5XFpOZWR6LyCnzY1muxVv1/ZjcXMIeV09sPiiiDrOnjpR6ihAZdnIjMd6Osfgh3oBK16FctUk+BE0cPYMWMvwSfQvbUFbgwkF9Zdo3iZltjWD/9nyJ9y08joIgoC93R+grb8TANDW34kT/V1wWeU/VkaJ0wtIHgxXRLOYvJaF5KPVVEY2LKD3dC/CN//6/0JkdO5DjfPMUbz9t9tQVTK+XkkoGO9QXlQM9BxoVWU6cfIxNEXB/InbJ9ZOTSIUjU/5BS8cQU+dS7Upv2QSrRm8uHJtxn+tUXIMV0TTWe3A3AfDUxq0msrIhgX0gT5LwmAFAKOxXLy/ugjL3SHk+IeAz9ZuSd4zKK/KRW14AB17AKG2EqiWf4y+Pc1np/fOG5/em2zq+ilMjA+Abhr7JlszKElSxn+tUXIMV0SkKrWnMrRcQK9HrurPo9YdAiblFG+wBT2+TuDNT1DR24NwjwvB6tUIfNwCkyjPv43U7UEtunQ1vbcQydYMPrr/3/i1RgxXRJTZ2AsoucnH7ki9vVjdOYYeAJXhT2CW6fDqEVMXghX98KzXz/TefCVbMwgAHaHuGbfxay37MFwRzUHwh4GKIq2HoTojtyuYjr2A5qfWvQleRws+QhOAtRhe6oFZisny3CfzfRDKy3UzvbcQydYMJsKvtezCcEU0i/hC32yTCe0KJmMvoPlz2tzoPU9AuAPwrV0CwSRP5aoAxg1VcYnWDEZjUdz55gPoj8xcmA/way0uk355S4ThilLm8ZoRCI0vmBWlGLqjJRjJtcAkjN9WWhyDyynPb7mkDbXbFSj9RsteQAvjKFmGMCJwlCxLq/VGJkq0ZvDlv35y4mtteq+rn1z6/8JeYMvqr7VM++UtEYYrSonHa8a1t1UiEp18QOiSKfex5Ip4/ZkeBiyDStSuQKnXU+ONlr2ASC2Tv9am97oKRkI432Hsyl26Mr3X3GSm5HchAgIh87RgNVMkapqobJHxxN/44rucJi/CVfL1ACj6OtmitCwCS17iX2wseTGUlkVUGlH2mvyLCjC1TYPc9vU04a9e+Vvs62mS/bnlpOa/SaoiA32KPTcrVzKJ9A0AhTath0EyM/vCQBZ8WpO1K9hwjbxTdtnQ1FNtrqoRvP7/vYNA39zTTqVlEbiqZnZoJ3mp1VfNSNNseus1Fwn5k98pDQxXRHOx2QAEtR6FKlJpV+CCfJ279fZGmylcVSMMTxpTs6+aUabZ9NprTiotU+y5OS1IlOUmtyuYjQABvzj0a9nK99OnB+L0ME1AxqO3abHp0+txck+z63GabS5q/ZukSumqFcBwRZT1UmlXcHrwDMakMVleT29vtGRc06fFtA4WqfyiIlcAUnuN5EKp+W8yr3Ettiv6/JwWlFHEH4LFXqz1MChNns4xSIPDkLo6MWILw+soMnx/nkRSaVdgs5RgqC037ddiU0+Sk96mxdTqq6bXabbZ6K3XnBpVK4DhSjZSWRkwIs9v9qQdu7sUfgCnOy9FxaF34CzpR6/3XXgvQ0YHrGTtCsSYhBNIf5eZ3t5oybj0uClCrb5qRjrSSY+95pSuWgEMV5Si0uIYLLliwnYMllwRpcXG73E1OWBZu1tRjoPoPnwAHeWdqF76Ra2HZ2h6fKMlY9Lrpgil+6oZsfqrl15zkZBflWAFMFxRilzOGF5/pmdah/ZWVOUuz8gO7XZ3KeAuhb9lCQrf7cfSwRCkNcBJ6c8ANmo9PEPTyxutUWXL8SGJGGlaTG6s/i6MWtOBcQxXlDKX82x4EqUYFkX6sdQSmQhXmcjuLkVgYCOsJVEU9b6PmiXlOKH1oChrGamvkZKMNC0mN1Z/F06tqhXAcCU7LmonIqXobQG3Fow4LSY3Vn/nR+2qFcBWDLKS7PxiJyJlGKmvkZLmMy1GFKdm1Qpg5YooNf3Z0amd9EuvC7jVxmkxmg81F7FPxnBFlIRodQIDPq2HQVksmxdwz4bTYpQKLaYD4zgtqICIP6T1EIgog7CrPdHCaFG1AhiuZMd1V0Tp0dtZcVrT6/EhRHqmZdUKYLgiIh3R21lxesAF3ETzEw9WWlWtAK65IiIdYauBmbiAm2j+tAxWAMMVEemEHs+K0wsu4CZKjVa7A6fjtKBCuKidaH6mL9rmYm0img+t11lNxnClAC5qJ5qf6Q0y47K1USbJg5sjsoce1llNxnBFRJpjqwGSGzdHZB+9BCuA4YqINMZWA6SE2TZHUGbS03RgHMOVgrjuiig5thogufEcxuyht+nAOO4WVIhkd0Dw88gUomTYaoDkxnMYs4NegxXAcEVEOsBWAyQXnsOYXfQYrABOCxIRkQK02qnHzRHZQS/9rObCcEWUCqsdUnhI61FQljB6CwGtdupxc0R20OMC9ukYrhQk2R1c1J4pBvwQigq0HgVlgUxoIaDVTj1ujsh8el5nNRnXXBER6Yic5yvu62nCo+8/hXs23IGNlRfKOcw5aXmMETdHZDajBCuA4YqISDfkDCbTK2Abrr9AlYXcWu/U4+aIzGSkYAVwWpCISFWJ1lPJeb6iFlNzPMaIlGC0YAUwXKmC666ICEi8nkrOYKJVE03u1COlGClYAQxXiuMhzkQUl6iaJGcwkbMClqps26ln9B2dRqH3lgtzYbgiIsMy0g+4RNUkOYOJVlNz2bRTLxN2dBqBUYMVwAXtRKSRdHeyabVge6ESLfReV9GQcjBJttNt+uvM9npKLCzPpp16cu7opNkZoZdVIgxXRKQ6OYKRkX7AJTuS5cXrt8sSTCZXwGYLavEKmFJtEZTYqadFO4lEtGw1kS2MuIB9Ok4LqoDNRImmSncnm1YLthcqlfVULqsD5y1ePud/FYXJQ0umTc3pcfpNi/Vs2SQTghXAyhURqUyO3/y17qU0H2pWkzJtak5v1UkeCq2sTAlWAMMVEaks3WBktB9w86kmyRF6MqWJph6n37Raz5YNMilYAQxXRKQiOYLRM82/N9QPuEyrJqklnRCuxDotrdezZbJMC1YAwxVRUqYBr9ZDyBjp/uYviiL+/dALc35crz/gMqWapJZ0QrhSu0jVrkBmi0wMVgDDlWrGF7X7YLEXaz0UWogSGxDUehDGJsdv/m93H8BIbHTu1+APuIyQTghXap0WK5Dyy9RgBTBcEZFK0v3NX5IkPNX024lwJkDAOSVL8OhlP5jSfJM/4IwtnRCu9DotViDlY+QGoalguCIiVaT7m//0aoYECSf6uxAcDelqjRWlJ50QbqRdpNkqk6tVkzFcEZFqFvqbv9F2CNLCLTSEa/k1ordGp3qVLcEKMHAT0YGBAXz/+99HZWUlFi1ahAsuuAAvvvhiSo9tbGyEIAiz/nf69GmFR05G4m8JQOr2QPzk/6AXHnSaT2k9pKwk56HGpH8Laaiq1deIGo1OjXSG5lyyKVgBBq5c3XDDDThw4AC2bduGFStW4Pnnn8fNN98MURRxyy23pPQcO3bswMqVK6fcZrcr94nnonZj8bcEUPjuLhQW9mJ4eQSe9aXIty9BeFDrkWUXboGnZLT8GlG60anRztCcTbYFK8Cg4Wr37t3405/+NBGoAODKK69ER0cHfvCDH+DGG2+E2WxO+jz19fW46KKLlB4uGZBvTzOsva2oWBrCoYYBCOXlqHVvghiTEEZE6+FlFW6Bp2S0+hpRo9Gp3rrUz1c2BivAoOHq1VdfhdVqxZYtW6bc/u1vfxu33HIL9u/fj4svvlij0ZGR+VsCEDtPoWLwHQy7gzhcGkRB/SVw2txaDy1rcQs8JaPV14jSC+j12KV+PrI1WAEGDVeHDx/GqlWrkJMzdfirV6+e+Hgq4eq6666Dz+dDSUkJrrjiCjz88MOor69P+JjR0VGMjp7tsxMKjR/ILEoxiNJY0tcUIEKUYknvp3fxa8iEa5lMEkQ4a80o7C/E2IV1WGTpR1nRcoix8d+Ip/8/W2h93eX5i1GevzjhfZQYm9bXrRUjXrccXyPzue5kC+g3lF+YdgDa2/3BrOHt3a4PZK1eyf35joT7AABSWdn4Df9/e/ceHFV59wH8e3ZDbuQCCUnI0lyGCsGEXIpvJbxSFBNsLJ2W4KRXFCo4Y6edlqEzTAWBpNSRcXRspmVsCQhpA9HyaoYZC602TvpKEEUMvhusApoQCgpxw+ZqLpt93j/ixiy7SfZyds/t+5lhnBzOZn9Pfgt+Oed5nuNQ5+colJ9vTYYrm82G+fPnexxP+qKRNpttytfPnTsX27dvR3FxMRISEmC1WrFnzx4UFxejubkZhYWFk772ySefRFVVlcfx6yNW9A7HTl98HKCnu0rtI+8rXYK8Mr7YKzSjEBgBMGKB/f88G9Z+fiTclakCx20sHPfkWnpaptzo9OXmt/G1hMBXDgoh8MyFWphgghMTwhtMeOZULeYuXCz71Sv5+h039p82df/PbmAgdJ9vxcNVU1MTVq5c6dO5LS0tKCoqAoApP1TTfeDKyspQVlY2/vWKFSuwevVq5OfnY+fOnTh27Nikr33sscewZcuW8a97enqQkZGBtBn5SIic5dM4pC4bIpPifTpXrZxiFO0j7yN7Ri5M0vTz27Si6yM7UnAd8d1n0D0/ErYUIOmrX4Zt56hA+/kRZOfNgMms/svycuG4Oe6JTl9rwZ4zf8Svv/4oinW09YCv/RZCYPvxI1NOoP+f7iNYe9edAQeg5qtncem9S541wolLn1/Cp3NaZbt6Jdfn3OOKlcr12HUcrnJyclBTU+PTuZmZmQDGVvR5uzrV1TXW2KQAGpudnY3ly5fj9OnTU54XFRWFqKgoj+MmyQyT5NuPU4JJN4FkbNz6GAsASMIEMySYnU6Y4YQzNd7rXzYms2So/9m6cNzy0Mq+SG9fP4c9Z/7oVqcQAtUth/Bx9xVUtxzCsq8Ef/tLbabr9/CobxPoRyVHQPO8hBD4w7k/Txne/nDuz1iecYesP/tgPufDPTaYTGPzq7TyaQjl32WKh6v09HRs2rTJr9fk5+ejvr4eDofDbd6V1WoFgGnnTU1GCAGTSbNbfxGRBmhlaf3EEDWxTq2vXpNDqCfQa2mFrJEnrU9F8XAViPLyctTU1OCll17C97///fHjtbW1sFgsWLp0qd/fs62tDc3NzSgtLZWzVCIiN1oJJ+d6z3nU+d+WJZpevSanUD5nUCsrZBmsJqfJcHX//fdj1apV+OlPf4qenh7cdtttqK+vx9///nfU1dW57XG1ceNG1NbW4qOPPkJWVhYAoLS0FCtWrEBBQcH4hPannnoKkiRh9+7dSg2LVEQM9ALgXxgkL60srRdC4PAnhz3qFELw2X1hovaHRDNYTU2z98BefvllPPjgg9i5cyfKysrw1ltvob6+Hj/+8Y/dzhsdHcXo6KjbIwny8/Px4osv4qGHHsI3v/lNPPXUU7j33nvxzjvvBHxL0V/Dtp6wvA8FTiRre9GBkan1cSG3PqJFrY/vOXXtXVz6/JJHnU+efg4myf1/GxODF+nfcI8Nwz02iDnJDFZT0OSVKwCIi4tDdXU1qqurpzzv0KFDOHTokNuxZ599NoSVTU8kp0CydSpaA5FeqXVOk1YePu2aTH3rFgASJFzuveZxPq9eGQevVvlOs1euiIi88TanSQ208vDp8TrhXudkk6uBL5/dx6tX+uS6WgUwWPmK4YroVn02SPE+bAhLqjPx6hCgnltWEx8s7I1awsl0dU76ugmr10hfJoYqBivfafa2IBHRrUL9rLdAaWVp/XR1AsCsqHjsLanCDPMMt+NqWL1G8nGFKoBXqwLBcEVEuqDmOU1aWVrvqtM2YMfViyOYt2AGTCb3n1lSTCLmzlTvKjYKHm8BBo/himgCU98NpUugAN161cpFLVevplpa/+a1Fvz6f59SxY7t6XEpSIuZg+irw5ifHGnIHfmNari3C659tBmsgsM5VwoRySncjkGtEn17RiSph1bmNHlz6+pGNdZIxsG5VfJguCL6gu3iTcScexPOD16HPW0Q7eYOpUsiH/kzp0lt1Lq6kYxhuMemuQcuawFvC5Lh2S7ehLPjP4jvOAvz/Iv4ON8EKS0NWQtKlC6NfKSVOU230sqO7aRP43OrkpKAtmGFq9EXhisyNFewmtv/BgbnX8W10nTEpmQiddYCpUsjP6n9cSHeqHV1I+mbx4R1B29Fy423BcmwJgYrZ5GEG19LRAyDFYXJrXtyuahlby7SH24GGj4MV2Ro6ZkRSEiPgRSfgOjkeQxWFDZa2bFdLdT6vEgtuDVUMViFHsMV0RdGU/igZgoPLa9uVAJXVAaOoUoZDFcK43YMRMaj5dWNSuCKSv+5rlYxVCmDE9oVJJJTINk6lS7D8MRAL0RyttJlkIFodXWjErii0j98bI06MFwRESlAi6sblcAVlb5hqFIX3hYkY+uzTX8OESmCKyqnx8nq6sRwRYYnxccqXQIRecEVlZNjqFI3hisiIlIdrqj0jqFKGzjnigzL1HdD6RKIaBL+rKg0wsR/zqnSFoYrMrbEWYBd6SKI6FZcUTmGoUqbGK7IkCT7daVLIKJpGHlFJUNVaPUN2jAwNBCy789wpQLDth5EJicoXYah2E7+GzH9n8EZfw0txZ2QzIOIQabSZRGRwTFUhV7f4NjPeEZCUsjeg+FKYdxINLxcD2vOwhXYF7Tj4+xhxC7+Op8pSESKYqgKPVeoikwc+/kO3ewP2XsxXJFh2C7eROyZJsRFXID9tmF8snQ2YlMyGayISDEMVeFxa7AKNYYrMgTXFau4uE/g/KqET4pmI4bBiogUwEAVXuEOVgDDFRlIemYE4q450GOZh5iUeAYrIgorhqrwUyJYAQxXZEAiOV7pEojIQBiqwk+pUOXCcEXGwecIElGYMFApR+lgBTBckcHwOYJEFEoMVcpSQ7ACGK6IiIiCwkClPLWEKheGKzIEU98NPuqGiGTFUKUOagtWAMMVERGRzxio1MMVqgB1BSuA4YoM4GbLh4g59yacaT2wF5rQbu7go26IyGcTAxXAUKUGarxaNRHDFelaZ6MV8R1nkV7oQEt2J6S0NGQtKFG6LCJSOQYqdVLz1aqJGK5Il8afIfj5OXTPv4iWbBOfIUhEPhnu7YLJxEClNmq/WjURwxXpjitYze1/A92JV3GtNJ3PECSiSbmuUjmdABAHkZQEESEpWhO501KwAhiuSKfSMyOQLBLRmZLIR90QkQdvE9OFQwBtw0qVRF5oLVS5MFyRrvFRN0TkwpV+2qGVuVWTYbgifeqzATOVLoKIlMRJ6dqk1atVEzFcke44O/6DyM/bYFqayG0XiAyGgUq7tH61aiKGK9KN8YnstuPoTuvB+2YTYlO4QpBI7xiotE8PV6smYrgiXbBdvInYM02Ii/sEg2k9XCFIpHMMVPqgp6tVEzFckeZ1NloRd/0SLAvssM5uh5Q6hxuFEukMw5S+6DVUuTBckabZLt5ENtoQucAOe9ogYr5aiOQFS5Qui4hkwEClT3q7BegNwxXpQvRXUiFF25Uug4iCwDClb3q/WjURwxXpymgK97Ui0opbwxTAQKVHRgpVLgxXpAvOXjsQrXQVRDQVhinjMcItQG8Yrkg3uBs7kfrwVp8xGTVUuTBckeaJz7sBmJQug4jAMGV0RrwF6A3DFemCaTavWhEpgWGKAIaqWzFckaaZ+m4oXQKRoTBM0UQMVd4xXJEmuR51E99xFvbCK3jfPAzJnIasWdw8lEguw71dMN1yx51higCGqukwXJHmTHyG4OeFw7iWPYzYxXyGIFGwXFelnE4AiAPAMEWejD5Z3RcMV6QpE58h6MyT8EnRbD5DkChAk22NIBwCaBuGSEqCpEBdpE4MVb5juCLNcAUrywI77LF9aC+K5TMEifzA+VIUCIYq/zFckabMSTMjMjESUso8xHA3dqJJccNOChZDVeAYrkiTuGEokTuGKZILQ1XwGK5IU7hhKBGDFIVG/1AXJAdDlRwYrkgTXCsEhwau4fOZM9FuvokYZCpdFlFYMExRqPQN2iBGASAOMxKSYIrgEgY5MFyR6nU2WhF3/RLiIi7g8hIbRrNnIoYrBEnHGKYo1CbuUzUjIQnAsHLF6BDDFamW62pV3PVLMC/4Ny7PtmP09kyuECRdYZCicPK2+afTIZQqR7c0OXmlt7cXW7duxX333YeUlBRIkoTKykq/vseNGzewYcMGzJkzB7GxsVi2bBkaGxtDUzD5bXyj0P43YF7wb3TmRCDm9kIGK9K84R6b2y/gi72lJvwiklvfoM1tojrnVYWWJsOVzWbDvn37MDQ0hDVr1vj9+qGhIZSUlKCxsRHV1dU4duwY0tLSUFZWhn/9618hqHhykq0zrO+nJdloQ+rXsxD9X4WITp6H5AVLlC6JyG8MU6Qkhirvupw22J1dIfv+mrwtmJWVhZs3b0KSJHz22WfYv3+/X68/cOAAWltbcerUKSxbtgwAsHLlShQWFmLr1q146623QlH2pCKTE8L6fppjt2M0j1svkPrxFh+pAZ/7N7ku59jPZmZ0MkRUdMjeR5PhSpKCW83Q0NCAnJyc8WAFABEREVi3bh22bduGq1evYt68ecGWSTJw9tqB0H3+iYLCMEVqwlA1OVeoAsaCVahpMlwFq7W1Fd/4xjc8jhcUFAAAzp8/P2m4GhoawtDQ0PjX3d3dAIDe/u6AapEGehAZ7QzotUpyCicGRgbQM9INkyT/3eW+wR7YHYMYGRyGPWom+noGEY1+2d/HX85RgYGBEfTYR2AyG2fJMsf95biH+9xvJYjZSe4vuqn85zRY7Le2xj0w9OVncmzlHzDkx+fQ6Rgbd+/NEV1uxeC6/RcbNfaz6RsY+9n028f+K4T8E/oNGa5sNhuSkpI8jruO2Wye/xp1efLJJ1FVVeVx/Dvrl3k5m4iIiNTMZrMhMTFR1u+peLhqamrCypUrfTq3paUFRUVFsrzvVLcWp/q9xx57DFu2bBn/2m63IysrCx0dHbI3R816enqQkZGBK1euICHBOHPGOG6O2wg4bo7bCLq7u5GZmen1YkuwFA9XOTk5qKmp8enczEx5duROTk72enWqq2vs0uFUP+ioqChERUV5HE9MTDTUh9IlISGB4zYQjttYOG5jMeq4TSb5p7YoHq7S09OxadOmsL5nfn4+rFarx3HXscWLF4e1HiIiItIPTe5zFazy8nJ88MEHblsuOBwO1NXVYenSpbBYLApWR0RERFpmrvR3a3OVOHHiBN577z1YrVYcO3YMqampkCQJ77//PrKzszFjxgwAwMaNG/HAAw9g/fr1mDVrFoCxVYENDQ2oq6tDWloarl+/jq1bt+LUqVP4y1/+guzsbL9qMZvNuOeeexARofiFwLDiuDluI+C4OW4j4LjlHbckQrEGMQyys7Nx+fJlr7/X1tY2HpA2bNiA2tpat2MAxgPVK6+8goGBARQVFWH37t0oLS0NQ/VERESkV5oNV0RERERqZMg5V0REREShwnBFREREJCOGKz/09vZi69atuO+++5CSkgJJkuDPeoBDhw5BkiSvvz799NPQFR6kYMcNADdu3MCGDRswZ84cxMbGYtmyZWhsbAxNwTLq6+vD5s2bYbFYEB0djaKiIrzwwgs+vVbt/Q5mbFrtJxD4uNXez+kE++dYqz0PZtxa7vnrr7+Ohx9+GIsWLcLMmTMxb948fPe738XZs2d9er1W+x3MuOXst7GWBQTJZrNh3759KCwsxJo1a7B///6Avs/BgwexaNEit2PJyep9yGaw4x4aGkJJSQnsdjuqq6uRmpqKvXv3oqysDP/85z9x9913h6jy4K1duxZnzpzBnj17sHDhQhw5cgQ//OEP4XQ68aMf/cin76HWfgc6Ni33Ewi+p2rt53SC+XOs5Z7L8fe2Fnv+3HPPwWaz4Ze//CVyc3PR2dmJZ555BsXFxfjHP/6Be++9d9LXarnfwYzbRZZ+C/KZ0+kUTqdTCCFEZ2enACB27drl8+sPHjwoAIgzZ86EqMLQCHbce/fuFQDEqVOnxo+NjIyI3Nxcceedd8pdrmz+9re/CQDiyJEjbsdXrVolLBaLcDgcU75ezf0OZmxa7acQwY1bzf30RTB/jrXc82DGreWeX79+3eNYb2+vSEtLEyUlJVO+Vsv9DmbccvabtwX94Lo8aDTBjruhoQE5OTlYtuzLh1tHRERg3bp1ePvtt3H16lU5ypRdQ0MD4uLiUFFR4Xb8Jz/5Ca5du+a2Ca3WBDM2rfYT0HdPpxPMn2Mt99yof2+npqZ6HIuLi0Nubi6uXLky5Wu13O9gxi0nhisFfPvb34bZbEZSUhLWrl2L1tZWpUsKqdbWVhQUFHgcdx07f/58uEvySWtrK26//XaPzeVcdfvaNzX2O5ixabWfgDw9VWM/Q03LPZeDXnre3d2Nd999F3l5eVOep7d++zpuFzn6zTlXYTR37lxs374dxcXFSEhIgNVqxZ49e1BcXIzm5mYUFhYqXWJI2Gw2rw/Ddh3z9hBtNbDZbJg/f77HcV/rVnO/gxmbVvsJBDduNfcz1LTc82Dorec/+9nP0N/fj+3bt095nt767eu45ey3YcNVU1MTVq5c6dO5LS0tKCoqCvo9y8rKUFZWNv71ihUrsHr1auTn52Pnzp04duxY0O8xHSXGDWDKy/LhuGQf6LiDqVsN/Z5KMGNTup/BCLR2tfcz1LTc80Dpqec7duzA4cOH8fvf/x533HHHtOfrpd/+jFvOfhs2XOXk5KCmpsanczMzM0NWR3Z2NpYvX47Tp0+H7D0mUmLcycnJXv+l09XVBQBe/4Ukt0DGHYq6w93vyQQzNjX0M1By166WfoaalnsuNy32vKqqCr/97W/xxBNP4Oc///m05+ul3/6O25tA+23YcJWeno5NmzYpXQYAQAgBkyk809+UGHd+fj6sVqvHcdexxYsXh7yGQMadn5+P+vp6OBx3vCXRAAAGFUlEQVQOtzk6wdYdzn5PJpixqaGfgQpFT9XQz1DTcs9DQUs9r6qqQmVlJSorK7Ft2zafXqOHfgcy7skE0m9tfDp0rK2tDc3NzSguLla6lJApLy/HBx984LYSy+FwoK6uDkuXLoXFYlGwusmVl5ejr68PL730ktvx2tpaWCwWLF261O/vqZZ+BzM2rfYTkL+naulnqGm553LTUs93796NyspKPP7449i1a5fPr9N6vwMdtzcB9zvozRwM5vjx4+Lo0aPi+eefFwBERUWFOHr0qDh69Kjo7+8fP+/hhx8WZrNZtLe3jx8rKSkRVVVVoqGhQTQ2Norf/e53wmKxiPj4eGG1WpUYjs+CGffg4KDIy8sTGRkZ4vDhw+K1114T5eXlIiIiQjQ1NSkxHJ+tWrVKzJ49W+zbt0+8/vrr4pFHHhEARF1dndt5Wuy3L2PTWz+FCHzcau+nL3z5c6zHngc6bi33/OmnnxYARFlZmXjzzTc9frnord/BjFvOfjNc+SkrK0sA8Pqrra1t/Lz169d7HNu8ebPIzc0V8fHxIiIiQlgsFrFu3Trx4Ycfhn8gfgpm3EII8emnn4qHHnpIJCUliejoaFFcXCxee+218A4iAL29veIXv/iFmDt3roiMjBQFBQWivr7e4zwt9tuXsemtn0IEPm6199MXvvw51mPPAx23lnt+9913TzrmiddV9NbvYMYtZ78lIYTw71oXEREREU2Gc66IiIiIZMRwRURERCQjhisiIiIiGTFcEREREcmI4YqIiIhIRgxXRERERDJiuCIiIiKSEcMVERERkYwYroiIiIhkxHBFREREJCOGKyIiIiIZMVwRERERyYjhioiIiEhGDFdEZCj9/f1IS0uDJEmYP38+RkZGvJ43ODiI5cuXQ5IkREVFoampKbyFEpFmMVwRkaHMnDkT27ZtAwC0tbXh0KFDHucIIfDggw+iubkZkiShtrYW99xzT3gLJSLNkoQQQukiiIjCaWhoCAsXLkRHRweysrJw4cIFREZGjv/+li1b8OyzzwIAnn76afzqV79SqlQi0iBeuSIiw4mKisLOnTsBAJcvX8bzzz8//nvV1dXjwWrz5s0MVkTkN165IiJDGh0dRW5uLi5cuICMjAxcunQJr7zyCioqKuB0OlFRUYEXXngBJhP/DUpE/mG4IiLDevHFF/GDH/wAALBx40YcPnwYg4ODWLFiBV599VVERUUpXCERaRHDFREZlhACS5Yswblz58aP5eXl4eTJk5g1a9akr6urq8Mbb7yBs2fPwmq1Ynh4GAcPHsSGDRvCUDURqR2vdxORYUmShEceeWT869TUVJw4cWLKYAUAjz/+OPbt24fLly8jPT091GUSkcYwXBGRYV28eBG7du0a/7q/v9+nW4H79+9He3s7Ojs78eijj4ayRCLSIIYrIjKkGzduoKysDJ999hmSk5MBjIWrJ554YtrXlpaWIisrK9QlEpFGMVwRkeH09/dj9erV+PjjjxEXF4dXX30Va9asAQD86U9/QkdHh8IVEpGWMVwRkaE4HA5UVFTgnXfeQUREBP76179iyZIlqKqqgiRJGBoaQlVVldJlEpGGMVwRkaE8+uijOHHiBADgueeew/333w8AKCgowAMPPAAAqK2txYULFxSrkYi0jeGKiAyjsrISBw4cAADs2LEDmzZt8vh9k8mE0dFR7NixQ4kSiUgHGK6IyBAOHDgwfrtv/fr1+M1vfuNxTl5eHr73ve8BAI4ePeq2/xURka8YrohI944fPz6+ZUJpaSlqamomPXfXrl0wm80QQmD79u3hKpGIdCRC6QKIiELtW9/6FkZGRnw6d9GiRXA4HCGuiIj0jFeuiIiIiGTEZwsSEflp//79OHnyJADAarXi3XffxV133YXbbrsNALBmzZrxfbOIyHh4W5CIyE8nT55EbW2t27Hm5mY0NzcDALKzsxmuiAyMV66IiIiIZMQ5V0REREQyYrgiIiIikhHDFREREZGMGK6IiIiIZMRwRURERCQjhisiIiIiGTFcEREREcmI4YqIiIhIRgxXRERERDJiuCIiIiKSEcMVERERkYz+H+WGAhUG7VPMAAAAAElFTkSuQmCC\n", "text/plain": [ "
" ] }, "metadata": {}, "output_type": "display_data" } ], "source": [ "def plot_predictions(clf, axes):\n", " x0s = np.linspace(axes[0], axes[1], 100)\n", " x1s = np.linspace(axes[2], axes[3], 100)\n", " x0, x1 = np.meshgrid(x0s, x1s)\n", " X = np.c_[x0.ravel(), x1.ravel()]\n", " y_pred = clf.predict(X).reshape(x0.shape)\n", " y_decision = clf.decision_function(X).reshape(x0.shape)\n", " plt.contourf(x0, x1, y_pred, cmap=plt.cm.brg, alpha=0.2)\n", " plt.contourf(x0, x1, y_decision, cmap=plt.cm.brg, alpha=0.1)\n", "\n", "plot_predictions(polynomial_svm_clf, [-1.5, 2.5, -1, 1.5])\n", "plot_dataset(X, y, [-1.5, 2.5, -1, 1.5])\n", "\n", "plt.show()" ] }, { "cell_type": "markdown", "id": "bca5e82a", "metadata": {}, "source": [ "# 4. Noyau polynomial\n", "L'astuce du noyau permet d'obtenir le même résultat que si vous aviez ajouté de nombreux attributs polynomiaux, même avec des polynômes de très haut degré, sans avoir à les ajouter. Il n'y a donc pas d'explosion combinatoire du nombre d’attributs car vous n'ajoutez aucuns attributs. La classe SVC implémente cette astuce.\n", "\n", "### Question:\n", "Tester l'astuce du noyau polynomila sur le dataset Moons pour deux valeurs de degrés 3 et 10.\n", "\n", "### Note:\n", "Si votre modèle souffre du sur-apprentissage, vous voudrez peut-être réduire le degré polynomial. Inversement, s'il est sous- apprentissage, vous pouvez essayer de l'augmenter. L'hyperparamètre coef0 contrôle la mesure dans laquelle le modèle est influencé par les polynômes de haut degré par rapport aux polynômes de bas degré." ] }, { "cell_type": "code", "execution_count": 54, "id": "0ae91df7", "metadata": {}, "outputs": [ { "data": { "text/plain": [ "Pipeline(steps=[('scaler', StandardScaler()),\n", " ('svm_clf', SVC(C=5, coef0=1, kernel='poly'))])" ] }, "execution_count": 54, "metadata": {}, "output_type": "execute_result" } ], "source": [ "from sklearn.svm import SVC\n", "\n", "poly_kernel_svm_clf = Pipeline([\n", " (\"scaler\", StandardScaler()),\n", " (\"svm_clf\", SVC(kernel=\"poly\", degree=3, coef0=1, C=5))\n", " ])\n", "..........................." ] }, { "cell_type": "code", "execution_count": 55, "id": "3068a266", "metadata": {}, "outputs": [ { "data": { "text/plain": [ "Pipeline(steps=[('scaler', StandardScaler()),\n", " ('svm_clf', SVC(C=5, coef0=100, degree=10, kernel='poly'))])" ] }, "execution_count": 55, "metadata": {}, "output_type": "execute_result" } ], "source": [ "poly100_kernel_svm_clf = Pipeline([\n", " (\"scaler\", StandardScaler()),\n", " (\"svm_clf\", SVC(kernel=\"poly\", degree=10, coef0=100, C=5))\n", " ])\n", ".............................." ] }, { "cell_type": "code", "execution_count": 56, "id": "c2be77ac", "metadata": {}, "outputs": [ { "data": { "image/png": "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\n", "text/plain": [ "
" ] }, "metadata": {}, "output_type": "display_data" } ], "source": [ "fig, axes = plt.subplots(ncols=2, figsize=(10.5, 4), sharey=True)\n", "\n", "plt.sca(axes[0])\n", "plot_predictions(poly_kernel_svm_clf, [-1.5, 2.45, -1, 1.5])\n", "plot_dataset(X, y, [-1.5, 2.4, -1, 1.5])\n", "plt.title(r\"$d=3, r=1, C=5$\", fontsize=18)\n", "\n", "plt.sca(axes[1])\n", "plot_predictions(poly100_kernel_svm_clf, [-1.5, 2.45, -1, 1.5])\n", "plot_dataset(X, y, [-1.5, 2.4, -1, 1.5])\n", "plt.title(r\"$d=10, r=100, C=5$\", fontsize=18)\n", "plt.ylabel(\"\")\n", "\n", "plt.show()" ] }, { "cell_type": "markdown", "id": "26a1703e", "metadata": {}, "source": [ "# 5. Noyau RBF gaussien\n", "Tout comme l’astuce des attributs polynomiales, la méthode des attributs de similarité peut être utile avec n'importe quel algorithme d'apprentissage automatique, mais il peut être coûteux en calcul de calculer toutes les attributs supplémentaires, en particulier sur de grands ensembles d'apprentissage. Encore une fois, l'astuce du noyau fait sa magie SVM, permettant d'obtenir un résultat similaire comme si vous aviez ajouté de nombreux attributs de similarité.\n", "\n", "### Question:\n", "Appliquer la classification SVM avec le noyau RBF gaussien." ] }, { "cell_type": "code", "execution_count": 57, "id": "bdf7bce7", "metadata": {}, "outputs": [ { "data": { "text/plain": [ "Pipeline(steps=[('scaler', StandardScaler()),\n", " ('svm_clf', SVC(C=0.001, gamma=5))])" ] }, "execution_count": 57, "metadata": {}, "output_type": "execute_result" } ], "source": [ "rbf_kernel_svm_clf = Pipeline([\n", " (\"scaler\", StandardScaler()),\n", " (\"svm_clf\", SVC(kernel=\"rbf\", gamma=5, C=0.001))\n", " ])\n", "................................." ] }, { "cell_type": "code", "execution_count": 58, "id": "970ba671", "metadata": {}, "outputs": [ { "data": { "image/png": "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\n", "text/plain": [ "
" ] }, "metadata": {}, "output_type": "display_data" } ], "source": [ "from sklearn.svm import SVC\n", "\n", "gamma1, gamma2 = 0.1, 5\n", "C1, C2 = 0.001, 1000\n", "hyperparams = (gamma1, C1), (gamma1, C2), (gamma2, C1), (gamma2, C2)\n", "\n", "svm_clfs = []\n", "for gamma, C in hyperparams:\n", " rbf_kernel_svm_clf = Pipeline([\n", " (\"scaler\", StandardScaler()),\n", " (\"svm_clf\", SVC(kernel=\"rbf\", gamma=gamma, C=C))\n", " ])\n", " rbf_kernel_svm_clf.fit(..., ...)\n", " svm_clfs.append(rbf_kernel_svm_clf)\n", "\n", "fig, axes = plt.subplots(nrows=2, ncols=2, figsize=(10.5, 7), sharex=True, sharey=True)\n", "\n", "for i, svm_clf in enumerate(svm_clfs):\n", " plt.sca(axes[i // 2, i % 2])\n", " plot_predictions(svm_clf, [-1.5, 2.45, -1, 1.5])\n", " plot_dataset(X, y, [-1.5, 2.45, -1, 1.5])\n", " gamma, C = hyperparams[i]\n", " plt.title(r\"$\\gamma = {}, C = {}$\".format(gamma, C), fontsize=16)\n", " if i in (0, 1):\n", " plt.xlabel(\"\")\n", " if i in (1, 3):\n", " plt.ylabel(\"\")\n", "plt.show()" ] }, { "cell_type": "code", "execution_count": null, "id": "89d589de", "metadata": {}, "outputs": [], "source": [] } ], "metadata": { "kernelspec": { "display_name": "Python 3 (ipykernel)", "language": "python", "name": "python3" }, "language_info": { "codemirror_mode": { "name": "ipython", "version": 3 }, "file_extension": ".py", "mimetype": "text/x-python", "name": "python", "nbconvert_exporter": "python", "pygments_lexer": "ipython3", "version": "3.7.13" } }, "nbformat": 4, "nbformat_minor": 5 }